Retour

AnnalesVergesTES99.tex

Télécharger le fichier Fichier PDF
\documentclass[12pt]{article}
\usepackage[applemac]{inputenc} %pas nécessaire avec mllatex ou frlatex
\usepackage{amsmath,amssymb,yhmath,makeidx}
\usepackage[normalem]{ulem}
\usepackage{fancybox}
\usepackage{array}
\usepackage{ulem}
\usepackage{graphics,graphicx}
\usepackage{pstcol,pst-text,pst-char,pst-plot,pst-grad}
  \definecolor{violet}{rgb}{0.5,0,0.2}
   \definecolor{orange}{rgb}{0.8,0.5,0}
   \definecolor{darkred}{rgb}{0.8,0,0}
\setlength\paperheight{297mm}
\setlength\paperwidth{210mm}
\setlength{\topmargin}{0cm}
\setlength{\oddsidemargin}{1cm}
\setlength{\evensidemargin}{1cm}
\setlength{\hoffset}{0cm}
\setlength{\voffset}{-2,5cm}
\setlength{\topskip}{2cm}
\setlength{\footskip}{2cm}
\setlength{\textheight}{25cm}
\newcommand{\R}{\mathbb{R}}
\newcommand{\N}{\mathbb{N}}
\newcommand{\D}{\mathbb{D}}
\newcommand{\Z}{\mathbb{Z}}
\newcommand{\Q}{\mathbb{Q}}
\usepackage{french}
\begin{document}
 
\begin{document}
 
    \noindent \Large{\textbf{BACCALAURÉAT SÉRIE ES SESSION 
1999}}\normalsize{}
\vspace{3cm}
\begin{itemize}
    \item Sujet 1 : France métropolitaine, juin 1999\dotfill 2\\
    \item Sujet 2 : Amérique du Nord, juin 1999\dotfill 
    8\\ 
    \item Sujet 3 : Guadeloupe-Guyane, juin 1999 \dotfill 12\\
    \item Sujet 4 : Centres étrangers, juin 1999 \dotfill 17\\
    \item Sujet 5 : Polynésie, juin 1999 \dotfill 21\\ 
    \item Sujet 6 : La Réunion, juillet 1999 \dotfill 26\\
    \item Sujet 7 : Asie, juin 1999 \dotfill 31\\
    \item Sujet 8 : France; session de remplacement, septembre 
    1998\dotfill 36\\
    \item Sujet 9 : Amérique du Sud, novembre 1998\dotfill 41\\
    \item Sujet 10 : Nouvelle-Calédonie, décembre 1998\dotfill 46\\
    \item Sujet 11 : Polynésie, session de remplacement, septembre 
    1998\dotfill 48\\
    \item Sujet 12 : Sportifs de haut-niveau, octobre 1998\dotfill 53\\
   \end{itemize}
\newpage
%%   Sujet 1 France Métropolitaine %%%%
%%%%                               %%%%
 
\noindent  \doublebox{TES   
\hspace{2.9cm}\Large{\textbf{Baccalauréat juin 1999}} 
\hspace{2.9cm}\normalsize{}}\\
 
\noindent \textbf{Exercice 1}\hspace{2cm}6 points\\
 
Le tableau suivant donne l'indice mensuel des dépenses 
d'assurance maladie d'août 1994 à juin 1995 (tendances observées à 
fin juillet 1995 - base 100, janvier 1990).\\
\begin{center} 
\begin{tabular}{*{7}{|p{1.6cm}|}}\hline
Mois & Août 1994 & Octobre 1994& Décembre 1994& Février 1995& 	Avril 
1995 & Juin 1995\\ \hline
Rang du mois $x_i$ & ~1 &~ 3 &	~5 &~	7 &~	9 &~ 11\\ \hline 
Indice $y_i$& ~123,4 &~ 125,9 &~ 127,5 &~ 127,9&~ 129 &~131,4 \\ \hline
\end{tabular} \end{center} 
\footnotesize{(Source : Département statistique de la Caisse 
Nationale de l'Assurance Maladie des Travailleurs 
Salariés).}\normalsize{}\\
 \textit{Pour tout l'exercice, les détails des calculs statistiques 
ne sont pas demandés. Les résultats seront arrondis avec deux 
chiffres après la  virgule.}\\
 On a représenté ci-après le nuage de points $M_i(x_i~;~ y_i)$ 
associé à la série statistique dans un repère orthogonal. G désigne 
le point moyen du nuage. On veut réaliser un ajustement affine de ce nuage de 
points.\\ 
 
\noindent $\triangleright~$\textbf{1)} Déterminer les coordonnées du 
point G et placer ce point sur le graphique.\\ 
 
\noindent $\triangleright~$\textbf{2)} Le modèle étudié dans cette 
question sera appelée « droite de Mayer ».\\
\textbf{a)} G$_1$ désigne le point moyen des trois premiers points du 
nuage et G$_2$ celui des trois derniers points.\\
Déterminer les coordonnées de G$_1$ et de G$_2$.\\
\textbf{b)} Déterminer l'équation réduite de la droite (G$_1$G$_2$) 
sous la forme $y = \textrm{A}x + \textrm{B}$.\\
\textbf{c)} Tracer la droite (G$_1$G$_2$) sur le graphique 
précédent.\\
\textbf{d)} En utilisant la calculatrice, déterminer la somme des 
résidus pour cet ajustement affine :
$$\textrm{S}_1 = \sum(y_i - \textrm{A}x_i - \textrm{B})^2.$$ 
 
\noindent $\triangleright~$\textbf{3)} Le deuxième modèle proposé est 
celui des moindres carrés.\\
 La calculatrice donne :\\
$\bullet $ l'équation de la droite D d'ajustement de $y$ en $x~ :~   
y = 0,71x +  123,26$~;\\
$\bullet $ la somme des résidus pour cet ajustement S$_2 \approx 1,7$ 
(arrondie avec un chiffre après la virgule).\\
\textbf{a)} Des droites D et (G$_1$G$_2$), quelle est celle qui 
réalise le meilleur ajustement affine ? Justifier.\\
\textbf{b)} Tracer D sur le graphique précédent.\\
 
\noindent $\triangleright~$\textbf{4) a)} Quels sont les indices 
mensuels que l'on pouvait prévoir en utilisant l'ajustement affine par la
méthode des moindres carrés (question \textbf{3)}) pour les mois cités dans
le tableau ci-dessous ?\\
\textbf{b)} Recopier le tableau ci-dessous et le compléter.
\begin{center} 
\begin{tabular}{|p{4cm} *{3}{|c}|}\hline
 Mois &   Nov. 1995 &  	Déc. 1995 & Janvier 1996\\ \hline
 Indices prévisionnels calculés par l'ajustement affine des moindres 
 carrés. & & & \\ \hline 
Tendances réellement observées. & 134,3 & 133,4 & 133,5\\ \hline 
\end{tabular}\\
\end{center}
\textbf{c)} Quel commentaire peut-on faire ?\\
\begin{center}
\begin{pspicture}(8,5)
\psgrid[subgriddiv=2,gridlabelcolor=white]
\psline{->}(1,0.5)(8,0.5) \psline{->}(1,0.5)(1,5)
\rput(1,0.2){\small{0}} \rput(1.5,0.2){\small{1}} 
\rput(2.5,0.2){\small{3}} 
\rput(3.5,0.2){\small{5}} \rput(4.5,0.2){\small{7}} 
\rput(5.5,0.2){\small{9}}
 \rput(6.5,0.2){\small{11}} \rput(7.5,0.2){\small{$x$}}
\rput(0.3,0.5){\small{123}}  \rput(0.3,1){\small{124}} 
\rput(0.3,1.5){\small{125}} 
\rput(0.3,2){\small{126}} \rput(0.3,2.5){\small{127}} 
\rput(0.3,3){\small{128}} \rput(0.3,3.5){\small{129}} 
\rput(0.3,4){\small{130}}
\rput(1.4,4.6){Indice} \rput(5.6,0.7){Rang du mois}
 \rput(0.2,4.5){\small{$y$}}
\psdots*[dotstyle=square](1.5,0.73)(2.5,1.8)(3.5,2.7)(4.5,2.9)(5.5,3.5)
(6.5,4.75)
\end{pspicture}   
\end{center}
 
\noindent \textbf{Exercice 1}\hspace{2cm}5 points\\
\textbf{(obligatoire)}\\
 
La courbe ci-dessous représente une fonction $f$ définie et dérivable 
sur [0~ ;~$+~\infty[$ dans le repère (O ;~ $\overrightarrow{\imath},~
\overrightarrow{\jmath}$) .\\
On note $f'$ la fonction dérivée de $f$.\\
La droite T$_{\textrm{A}}$ est tangente au point A d'abscisse 0.\\
La courbe admet une tangente parallèle à l'axe des abscisses au point 
d'abscisse 1.\\
Enfin, la fonction $f$ est croissante sur [1~;~$+~\infty[$ et sa 
limite en +~$\infty$ est $+~\infty$.\\
 
\noindent $\triangleright~$\textbf{1)} À partir des informations 
portées sur le graphique et complétées par les précisions précédentes
, répondre aux questions suivantes : \\
\textbf{a)} Reproduire et compléter le tableau ci-dessous :
$$\begin{array}{*{3}{|c}|}\hline
~~~~x~~~~ & ~~~~0~~~~ &~~~~ 1~~~~\\ \hline
f(x) &  &\\ \hline 
f'(x) & & \\ \hline
\end{array}$$
\textbf{b)} Donner le tableau de variation de $f$ sur [0 ;~$+~\infty[$,
complété par la limite en  $+~\infty$.\\ 
 
\noindent $\triangleright~$\textbf{2)} On considère la fonction $g$ 
inverse de la fonction $f$, c'est-à-dire $g = \cfrac{1}{f}$.
On note $g'$, la fonction dérivée de $g$. \\
\textbf{a)} Déterminer $g(0),~ g(1),~ g(3)$.\\ 
\textbf{b)} Quel est le sens de variation de la fonction $g$ sur [0 
;~$+~\infty[$ ? 
Justifier la réponse donnée.\\
\textbf{c)} Déterminer les valeurs $g'(0),~ g'(1)$.\\
\textbf{d)} Déterminer la limite de $g$ en + ~$\infty$.\\
 
\noindent $\triangleright~$\textbf{3)} On souhaite traduire 
graphiquement les informations obtenues pour la fonction $g$.\\
Tracer une courbe qui satisfait aux résultats obtenus à la question 
\textbf{2)} dans un repère orthonormal (unité : 2 cm) sur une feuille 
de papier millimétré ; le tracé des tangentes aux points d'abscisses 0 et
1 devra apparaître sur la figure.\\
\begin{center}
    \begin{pspicture}(-1,-2)(5,4)
	\psgrid[subgriddiv=1](0,0)(-1,-2)(5,4)
	\psline{->}(0,2)(1.333,-2) \psline{<->}(0.5,1)(1.5,1)
	\rput(0.2,2.1){A} \rput(1.5,-1.5){T$_{\textrm{A}}$}
	\pscurve(0,2)(1,1)(2,1.2)(3,2)(4,2.82)(5,3.63)
	\end{pspicture}
	\end{center}
 
\noindent \textbf{Exercice 2}\hspace{2cm}5 points\\
\textbf{(spécialité)}\\
 
L'espace est muni d'un repère orthonormal (O ;~$\overrightarrow{\imath},~
\overrightarrow{\jmath},~\overrightarrow{k}$) représenté ci-après.
Le plan (R) est représenté par ses traces sur les plans de 
coordonnées ; il a pour équation : $x + z = 2$.\\ 
 
\noindent $\triangleright~$\textbf{1)} On donne les points A, B, C 
définis par leurs coordonnées respectives : A(6 ; 0 ; 0),  B(0 ; 3 ; 0) et
C(0 ; 0 ; 6).\\
\textbf{a)} Placer les points A, B, C dans le repère (O 
;~$\overrightarrow{\imath},~\overrightarrow{\jmath},~\overrightarrow{k}$) et
tracer le triangle ABC.\\
\textbf{b)} Calculer les coordonnées des vecteurs 
$\overrightarrow{\textrm{AB}}$ ~et ~$\overrightarrow{\textrm{AC}}$.\\
\textbf{c)} Soit $\overrightarrow{n}$ le vecteur de coordonnées (1 ; 2 ; 1). 
Montrer que le vecteur $\overrightarrow{n}$ est normal au plan (P) passant par A, B et C.\\ 
\textbf{d)} Vérifier que le plan (P) a pour équation $x + 2y + z = 
6$.\\
 
\noindent $\triangleright~$\textbf{2)} On a placé dans le repère les 
points G, E et F à coordonnées entières.\\
Le point G est situé sur l'axe (O~;~$\overrightarrow{\jmath}$), le 
point E dans le plan (O ;~$\overrightarrow{\imath},~\overrightarrow{\jmath}$)  et 
le point F dans le plan (O~;~$ \overrightarrow{\jmath},~ \overrightarrow{k}$).\\
Le plan (Q) passant par les points G, E et F est parallèle au plan 
(0 ;~$ \overrightarrow{\jmath},~ \overrightarrow{k}$).\\
\textbf{a)} Donner l'équation du plan (Q).\\
\textbf{b)} Donner les coordonnées des points G, E et F.\\
\textbf{c)} Parmi les points E, F et G quels sont ceux situés dans le 
plan (P) ?\\
\textbf{d)} Quelle est la nature de l'ensemble des points M dont les 
coordonnées $(x ~;~ y~;~ z)$ vérifient le système :
$$\left\{ \begin{array}{r c l}
y &=& 2\\
x + 2y + z &=& 6
\end{array}\right.$$
\textbf{e)} Représenter cet ensemble sur la figure ci-dessous.\\
\begin{center}
    \psset{unit=4mm}
    \begin{pspicture}(34,21)
	\psgrid[gridlabelcolor=white,subgriddiv=1]
	\psline[linewidth=2pt]{->}(17,12)(19,12) 
\psline[linewidth=2pt]{->}(17,12)(17,14) 
	\psline[linewidth=2pt]{->}(17,12)(15.586,10.586)
	\psline[linewidth=2pt](0,12)(34,12) 
\psline[linewidth=2pt](17,0)(17,21)
	\psline[linewidth=2pt](15,10)(34,10) 
\psline[linewidth=2pt](17,16)(34,16)
	\psline[linewidth=2pt](15,10)(17,16)
	\psline[linewidth=2pt](17,12)(5,0)
	\rput(16.2,12.2){O} \rput(16,10.5){$\vec{\imath}$}
	\rput(18.5,12.5){$\vec{\jmath}$}\rput(17.5,13.5){$\vec{k}$}
	\rput(19,9){E} \rput(21,16.5){F} \rput(21,12.5){G}
	\rput(30,15){(R)} 
	\end{pspicture}
\end{center}
 
\noindent $\triangleright~$\textbf{3)} On considère le système S de 3 
équations à 3 inconnues $x,~ y,~ z $:
$$\left\{ \begin{array}{r c l}
x + z& =& 2\\
y &=& 2\\ 
x + 2y + z& =& 6
\end{array} \right. $$
Quel est l'ensemble des points de l'espace dont les coordonnées sont 
les solutions du système S ?
 
\noindent \textbf{Problème}\hspace{2cm}9 points\\
 
On a tracé dans un repère orthonormal (O ;~$ \overrightarrow{\imath},~
\overrightarrow{\jmath}$) la courbe représentative ($\mathcal{C}$) de la
fonction $f$ définie sur l'intervalle ]0 ; 4] par $f(x) = x - \cfrac{1}{2} -
\ln x$.\\ 
\textit{Dans tout le problème on donnera les résultats arrondis à} 
$10^{-3}$.\\ 
 
\noindent \textbf{A - Étude théorique liée à la fonction}  $f$\\
 
\noindent $\triangleright~$\textbf{1) a)} Étudier le sens de 
variation de la fonction $f$ sur l'intervalle ]0 ; 4].\\
\textbf{b)}  Étudier la limite de $f$ en 0.\\
\textbf{c)} Donner le tableau de variation de $f$.\\
 
\noindent $\triangleright~$\textbf{2)}	Soit (Z) la partie du plan 
délimitée par la courbe ($\mathcal{C}$) et les droites d'équations $y = 
\cfrac{1}{2},~x = 1$ et $x = 3$.\\
\textbf{a)} Justifier que l'on a $f(x) „ \cfrac{1}{2}$	sur ]0 ; 4] et 
exprimer à l'aide d'une intégrale (que l'on n'essaiera pas de calculer dans 
cette question) l'aire $\mathcal{A}$,  en unités d'aire, de la partie (Z) du
plan. \\
\textbf{b)} Soit $g$ la fonction définie sur ]0 ; 4] par $g(x) = x 
\ln x - x$.\\ 
Calculer $g'(x)$.\\
\textbf{c)} En déduire la valeur exacte de l'aire $\mathcal{A}$, en 
unités d'aire.\\
 
\noindent \textbf{B - Probabilité et jeu}\\
Au cours de l'élaboration d'une phase d'un jeu vidéo inspiré du golf, 
on cherche à évaluer la probabilité de gagner.\\
L'écran est le carré AOFB. Les sommets du carré ont pour coordonnées :\\
A(0 ; 4),  0(0 ; 0) F(4 ; 0) B(4 ; 4).\\
La courbe ($\mathcal{C}$) partage l'écran en deux parties :\\
$\bullet~$ la partie de l'écran située strictement au-dessus de la 
courbe représente une mare et elle est notée (M) ;\\
$\bullet~$ la partie de l'écran située au-dessous de la courbe 
représente le terrain de jeu et elle est notée (T).\\
La partie (Z) définie au paragraphe \textbf{A} est donc incluse dans 
(T).\\
 
\noindent $\triangleright~$\textbf{1)} Dans cette question, le jeu 
consiste à simuler le lancer d'une balle. On admet que la probabilité 
d'atteindre une partie de l'écran est donnée par :
$$\cfrac{\textrm{Aire de la partie de l'écran considérée}} 
{\textrm{Aire du carré AOFB}} $$
Cette probabilité est indépendante de l'unité graphique choisie.\\ 
Déterminer, par le calcul, la probabilité que la balle atteigne la 
zone (Z).\\ 
 
\noindent $\triangleright~$\textbf{2)} Dans cette question, le jeu 
consiste à simuler trois lancers successifs et indépendants ; on admet que,
pour chaque lancer, la probabilité d'atteindre (Z) est de 0,044.\\ 
On gagne lorsque deux au moins des trois balles lancées ont atteint 
la partie (Z).\\ 
Calculer la probabilité de gagner.\\
On pourra s'aider d'un arbre et on fera figurer le détail des calculs 
sur la copie.\\
\begin{center}
    \psset{unit=2cm}
\begin{pspicture}(-0.5,-0.5)(4,4)
    \psgrid[subgriddiv=1](0,0)(-0.5,-0.5)(4.5,4.5)
    \psline{->}(0,0)(1,0) \psline{->}(0,0)(0,1) 
    \rput(-0.3,0.3){O} \rput(3.5,0.2){(T)}  
    \rput(3.5,2){($\mathcal{C}$)}
    \rput(1.5,2.8){(M)} \rput(-0.3,4.1){A} \rput(4.2,4.2){B}
    \rput(4.2,-0.3){F} \rput(0.5,-0.3){$\vec{\imath}$}
    \rput(-0.3,0.5){$\vec{\jmath}$} 
\pscustom[linewidth=0.7mm]
{\pscurve(1,0.5)(1.25,0.527)(1.5,0.595)(1.75,0.69)(2,0.807)(2.25,0.939)
(2.5,1.0837)(2.75,1.238)(3,1.4014)
\gsave
\psline[liftpen=1](3,0.5)(1,0.5)%(3,1.40139)
\fill[fillstyle=solid,fillcolor=lightgray]
\grestore}
\psline[linewidth=0.7mm](1,0.5)(3,0.5) \psline[linewidth=0.7mm](3,0.5)(3,1.401)
\pscurve(0.05,2.546)(0.1,1.903)(0.2,1.309)(0.25,1.1363)(0.5,0.693)(0.75,0.538)
(1,0.5)
%(1.25,0.527)(1.5,0.595)(1.75,0.69)(2,0.807)(2.25,0.939)(2.5,1.0837)(2.75,1.238)
\pscurve(3,1.4014)(3.25,1.571)(3.5,1.747)(3.75,1.928)(4,2.113)
\rput(2.7,0.8){(Z)}    	
 \end{pspicture}   
\end{center}
\footnote{\scalebox{1 -1}{National juin 1999}}
\newpage
 
%%%%%%%%%%%%%%%%%%%%%%%%%%%%%%%%%%%%%%%%
%% Sujet 2 Amérique du Nord juin 1999 %%
 
\noindent  \doublebox{TES   
\hspace{2.9cm}\Large{\textbf{Baccalauréat juin 1999}} 
\hspace{2.9cm}\normalsize{}}\\
 
\noindent \textbf{Exercice 1}\hspace{2cm}5 points\\
\textbf{Commun à tous les candidats}\\
 
Une salle de spectacle propose, pour la saison, des abonnements pour 
4, 5 ou 6 spectacles.\\ 
Dans la population des abonnés, la répartition est la suivante :\\
* 43,5 \% ont choisi l'abonnement 4 spectacles,\\ 
* 33 \% ont choisi l'abonnement 5 spectacles,\\ 
* le reste a choisi l'abonnement 6 spectacles. \\
D'autre part, 65 \% des abonnés sont des jeunes de moins de 25 ans, 
et dans cette population, la répartition est différente :\\ 
* 40 \% ont choisi l'abonnement 4 spectacles,\\
* 40 \% ont choisi l'abonnement 5 spectacles,\\
* le reste a choisi l'abonnement 6 spectacles.\\ 
On interroge un abonné au hasard.\\
 On note A l'événement « L'abonné interrogé a moins de 25 ans ». 
Ainsi la probabilité p(A) de cet événement est 0,65.\\ 
€ On note B l'événement « L'abonné interrogé a choisi 5 spectacles 
».\\ 
€ Pour tout événement V, on note $\overline{\textrm{V}}$ l'événement 
contraire  de V.\\ 
 
\noindent $\triangleright~$\textbf{1) a)} Quelle est la probabilité 
que l'abonné interrogé ait 25 ans ou plus ?\\
\textbf{b)} Sachant que l'abonné interrogé a moins de 25 ans, quelle 
est la probabilité qu'il ait choisi 5 spectacles ?\\
\textbf{c)} Décrire l'événement (A $\cap$ B), et démontrer que la 
probabilité p(A ~$\cap$~ B) est égale à 0,26.\\
 
\noindent $\triangleright~$\textbf{2) a)} Démontrer que la 
probabilité p($\overline{\textrm{A}} \cap$  B) est égale à 0,07.\\ 
\textbf{b)} En déduire la probabilité conditionnelle de B sachant que 
A est réalisé.\\ 
 
\noindent $\triangleright~$\textbf{3)} L'abonnement pour 4 spectacles 
coûte 50 euros, celui pour 5 spectacles coûte 60 euros, et celui pour 6
spectacles coûte 70 euros. On appelle $X$ la variable aléatoire égale à la
somme dépensée par l'abonné interrogé.\\ 
\textbf{a)} Donner la loi de probabilité de X en complétant :
$$\begin{array}{| c | c | c | c |}\hline
x_{i} & ~~~~50~~~~	&~~~~60~~~~	&~~~~ 	70~~~~\\ \hline
p(X = x_{i}) & & & \\ \hline
\end{array}$$
\textbf{b)} Calculer l'espérance de $X$.\\
 
\noindent \textbf{Exercice 1}\hspace{2cm}5 points\\
\textbf{Uniquement pour les candidats ayant opté pour l'enseignement 
de spécialité}\\
 
L'espace est rapporté au repère orthonormal (0 ;~$\overrightarrow{\imath}  
,~\overrightarrow{\jmath} ,~\overrightarrow{k}$).\\
ABCDOFGH est un pavé défini par $\overrightarrow{\textrm{OH}} = 3 
\vec{\imath} ,~ \overrightarrow{\textrm{OF}} = 
4\overrightarrow{\jmath}$ et $\overrightarrow{\textrm{OA}} = 3\overrightarrow{k}$
.\\
Soit L le milieu de [CG]. \\
\begin{center}
    \begin{pspicture}(7,6)
	\psframe(0.8,0.8)(4.8,3.8) %CGHD
	\pspolygon(0.8,3.8)(4.8,3.8)(6,5)(2,5) %ABCD
	\pspolygon(4.8,0.8)(6,2)(6,5)(4.8,3.8) %CBFG
	\psline[linestyle=dashed](2,3)(2,5)
	\psline[linestyle=dashed](3,2)(6,2)
	\psline[linestyle=dashed](2,2)(0.8,0.8)
	\psline{->}(2,2)(1.6,1.6)
	\psline{->}(2,2)(3,2)
	\psline{->}(2,2)(2,3)
	\rput(2.2,2.2){O} \rput(1.7,5){A} \rput(6.2,5.2){B}
	\rput(4.7,4){C} \rput(0.6,3.7){D} \rput(6.2,2.2){F}
	\rput(5,0.5){G} \rput(1,0.5){H} \rput(1.7,1.8){$\vec{\imath}$}
	\rput(2.5,1.6){$\overrightarrow{\jmath}$} 
\rput(1.8,2.4){$\overrightarrow{k}$}
\end{pspicture}
\end{center}
 
\noindent $\triangleright~$\textbf{1)} On considère l'ensemble 
($\Pi$) des points dont les coordonnées $x,~ y$ et $z$ vérifient :
$4x- 3y + 8z - 12 = 0$.\\ 
\textbf{a)} Parmi les points A, B, O, G, H, L lesquels appartiennent 
à ($\Pi$) ?\\
\textbf{b)} Justifier que l'ensemble ($\Pi$) est le plan (BLH).\\
 
\noindent $\triangleright~$\textbf{2) a)} Donner les coordonnées d'un 
vecteur normal $\overrightarrow{n}$ au plan (BLH).\\
\textbf{b)} Soit ($\Delta$) la droite passant par A et de vecteur 
directeur $\overrightarrow{n}$.\\
Montrer que ($\Delta$) est l'ensemble des points $M$ tels que 
$\left\{ \begin{array}{l}
\overrightarrow{\textrm{AM}} . \overrightarrow{\textrm{NH}} = 0\\
\textrm{et}\\
\overrightarrow{\textrm{AM}} . \overrightarrow{\textrm{BL}} = 0.\\
\end{array}\right.$
En déduire un système d'équations caractérisant la droite 
($\Delta$).\\ 
\textbf{c)} Montrer que le point de coordonnées $\left(-~ \cfrac{48}{89}~ ;~ 
\cfrac{36}{89}~ ;~\cfrac{171}{89} \right)$ appartient à ($\Delta$) et 
à $(\Pi)$.\\
 
\noindent \textbf{Exercice 2}\hspace{2cm}5 points\\
\textbf{Uniquement pour les candidats n'ayant pas suivi l'enseignement de
spécialité}\\
 
On donne, dans un repère orthonormal (0 ;~$\overrightarrow{\imath} 
,~\overrightarrow{\jmath}$) du plan, la courbe représentative 
($\Gamma$) d'une fonction $f$,
définie et dérivable sur [0 ; 6].\\ 
Les points A$\left(\cfrac{1}{2}	~ ; 2\right),~ B\left( 4~;~\cfrac{1}{4}\right)$
et C(2 ; 1) sont des points de ($\Gamma$), et (T) est la tangente à ($\Gamma$)
en C.
\begin{center}
    \begin{pspicture}(10,4)
	\psgrid[subgriddiv=4]
	\psline{->}(0,0)(1,0) \psline{->}(0,0)(0,1)  \psline(0,2)(4,0)
	\rput(0.6,2.1){A} \rput(0.6,1.4){(T)} \rput(2.2,1.2){C}
\rput(4.2,0.3){B} \rput(-0.2,-0.2){O} 
\rput(0.5,-0.5){$\overrightarrow{\imath}$ } 
\rput(-0.5,0.5){$\overrightarrow{\jmath}$ }
\pscurve(0,2.5)(0.5,2)(1,1.6)(1.5,1.25)(2,1)(3,0.62)(4,0.25)(5,0)(6,0.25)
 
\end{pspicture}
\end{center}
\vspace{1cm}
\noindent $\triangleright~$\textbf{1) a)} Déterminer par lecture 
graphique le minimum et le maximum de $f$ sur [0 ; 6].\\
\textbf{b)} Déterminer par lecture graphique l'image par $f$ de 
l'intervalle [0 ; 2].\\
\textbf{c)} En utilisant le graphique, donner l'ensemble des 
solutions de l'inéquation $f(x) < \cfrac{1}{2}$.\\
 
\noindent $\triangleright~$\textbf{2) a)} On admet que (T) est 
parallèle à (AB). 
Déterminer alors $f'(2)$.\\
\textbf{b)} Déterminer l'équation réduite de (T), et celle de (AB).\\ 
\textbf{c)} Justifier à l'aide du graphique que, pour tout $x$ de 
$\left[\cfrac{1}{2}~ ;~ 4\right]$ on a : 
$$-~\cfrac{1}{2} x + 2 ¾ f(x) ¾	-~\cfrac{1}{2} x + \cfrac{9}{4}.$$
 
\noindent $\triangleright~$\textbf{3)} On pose I =$ 
\int\limits_{\frac{1}{2}}^9 f(x)\: \textrm{d}x.$ Déduire du résultat 
précédent
\textbf{2) c} que l'intégrale I est comprise entre $\cfrac{49}{16}$ 
et $\cfrac{63}{16}$.\\ 
 
\noindent \textbf{Problème}\hspace{2cm}10 points\\
 
Une entreprise envisage la fabrication d'un nouveau produit. Sa 
décision dépend des résultats de plusieurs études :\\
\textbf{Étude de la demande pour ce nouveau produit} : c'est l'objet 
de la partie \textbf{A}.\\
\textbf{Étude d'un coût moyen de production} : c'est l'objet de la 
partie \textbf{B.}\\
 
\noindent \textbf{Partie A}\\
 
Une étude a permis d'établir le tableau suivant oû, pour différentes 
observations, $x_{i}$ désigne la quantité de produit (en milliers 
d'unités) que la clientèle est disposée à acheter, et $y_{i}$ le prix de
vente (en francs) d'une unité : 
$$\begin{array}{| c| c | c | c | c | c | c |}\hline
x_{i}& 	~~1,5~~&  ~~	3~~&~~5~~	&~~8~~	&~~11~~ &~~12~~\\ \hline
y_{i}&	120&	110& 100 &	90&	80&	70\\ \hline
\end{array}$$
Ainsi, pour que la clientèle soit disposée à acheter 5\:000 unités, 
le prix de vente d'une unité doit être fixé à 100 F.\\ 
$\triangleright~$\textbf{1)} Représenter le nuage de points associé 
à cette série statistique.\\ 
Prendre 1 cm pour 1 millier d'unités en abscisse, et 1 cm pour 10 
francs en ordonnée.\\ 
\textit{Dans les questions suivantes, le détail des calculs 
statistiques n'est pas demandé; les résultats seront donnés à} 
$10^{ -~ 2}$ \textit{près}.\\ 
 
$\triangleright~$\textbf{2)} Donner le c¦fficient de corrélation 
linéaire de cette série statistique.\\
Un ajustement affine est-il approprié ? Justifier la réponse.\\
 
$\triangleright~$\textbf{3) a)} Donner une équation de la droite 
d'ajustement de $y$ en $x$, obtenue par la méthode des moindres carrés.\\ 
\textbf{b)} D'après ce modèle, comment faut-il fixer le prix de vente 
d'une unité si l'on veut pouvoir vendre un minimum de 6\:500 unités ?\\ 
 
$\triangleright~$\textbf{4)} On admet que le prix de vente d'une 
unité, noté PV, est une fonction de la demande $x$ (en milliers d'unités)
définie, pour $ x \in$ [2 ; 15], par : PV$(x) = -~4,33x+ 124,2$.\\ 
Représenter la fonction PV dans le repère utilisé dans la question 
\textbf{1)}.\\
 
\noindent \textbf{Partie B}\\
 
Le coût total de production (en francs) de $x$ milliers d'unités est,
pour $x \in $ [2 ; 15] :
$$\textrm{CT}(x) = 105\left[x+ 4 - 3\ln (x)\right]$$
et le coût moyen de production d'une unité est, pour $x \in $ [2 ; 15]
$$\textrm{CM}(x) = \cfrac{\textrm{CT}(x)}{1000x}$$
 
\noindent $\triangleright~$\textbf{1)} On note CM' la dérivée de la 
fonction CM.\\ 
Calculer CM'($x$) et démontrer que CM'($x$) a le même signe que
$\ln (x) - \cfrac{7}{3}$  pour tout $x \in$ [ 2 ; 15].\\
 
\noindent $\triangleright~$\textbf{2)} Résoudre sur l'intervalle [0 
;~ +~ $\infty$[ l'inéquation $\ln (x) - \cfrac{7}{3} „ 0$.\\
 
\noindent $\triangleright~$\textbf{3) a)} Étudier les variations de 
CM sur l'intervalle [2 ; 15].\\
\textbf{b)} Tracer la représentation graphique de CM dans le repère 
utilisé dans la partie \textbf{A}.\\
\textbf{c)} À l'aide du graphique, déterminer l'ensemble des valeurs 
de $x$ pour lesquelles l'entreprise peut faire un bénéfice. (On donnera la 
réponse sous forme d'un intervalle dont les bornes sont des entiers.)\\
\footnote{\scalebox{1 -1}{Amérique du Nord juin 1999}}
\newpage
%%%%%%%%%%%%%%%%%%%%%%%%%%%%%%%%%%%%%%%%%%%%%%%%%%%%
%% Sujet 3 Guadeloupe-Guyane-Martinique juin 1999 %%
\noindent  \doublebox{TES   
\hspace{2.9cm}\Large{\textbf{Baccalauréat juin 1999}} 
\hspace{2.9cm}\normalsize{}}\\
 
\noindent \textbf{Exercice 1}\hspace{2cm}4 points\\
\textbf{Candidats n'ayant pas choisi l'enseignement de spécialité}\\
 
\noindent Le plan est rapporté à un repère orthonormal, dont les 
unités sont 1 cm sur chaque axe. Construire ce repère sur votre copie en
plaçant l'origine du repère en bas et à gauche.\\ 
 
\noindent \textbf{Partie A}\\
 
\noindent \textbf{a)} Représenter la droite (D$_{1}$) d'équation $3x+ 
y = 30$, la droite (D$_{2}$) d'équation $x + 4y = 32$ et la droite (D$_{3}$)
d'équation $x + y = 10$.\\
\textbf{b)} Déterminer au moyen d'un calcul les coordonnées du point 
d'intersection I des droites (D$_{1}$) et (D$_{2}$).\\ 
\textbf{c)} Repérer graphiquement à l'aide d'une croix (« $\times$ ») 
les points du plan dont les coordonnées sont des nombres entiers positifs, $x$
et $y$, qui vérifient de plus les conditions : 
$$3x + y ¾ 30~ ;~ x + 4y ¾ 32~;~x + y „ 10.$$
 
\noindent \textbf{ Partie B}\\
 
\noindent Un artisan fabrique deux sortes de poupées : des petites 
poupées et des grandes poupées.\\ 
Les petites poupées nécessitent 3 heures de travail et les grandes 
poupées une heure seulement. L'artisan, avec ses ouvriers, peut travailler 30 
heures au plus par jour.\\ 
L'artisan ne dispose que de 32 mètres de tissu par jour. Il lui faut 
1 mètre de tissu pour habiller une petite poupée et 4 mètres pour habiller une 
grande poupée.\\ 
On désigne par $x$ le nombre de petites poupées et par $y$ le nombre 
de grandes poupées produites dans une journée. L'artisan s'impose de fabriquer 
au moins 10 poupées par jour.\\ 
On admet que les contraintes de l'énoncé correspondent aux conditions 
suivantes : 
$$\begin{array}{l l}
x ~\textrm{et}~ y~ \textrm{sont deux nombres entiers positifs}&	3x+y 
¾	
30~;\\
x „ 0 ~;&	x + 4y ¾	32\\
y „ 0~;&	x + y  „ 10.\\
\end{array}$$
Le nombre total de poupées produites dans une journée de travail est 
représenté par $S = x +y$.\\ 
L'artisan veut que sa production journalière $S$ soit maximum.\\
Combien de poupées de chaque sorte doit-il fabriquer ?\\
 
\noindent \textbf{Exercice 1}\hspace{2cm}4 points\\
\textbf{Candidats ayant choisi l'enseignement de spécialité}\\
 
Une suite réelle ( U$_{n})_{n \in \N}$ est définie par son premier 
terme U$_{0}$ strictement positif et par la relation de récurrence 
suivante : 
$$\textrm{U}_{n+1} - \textrm{U}_{n} = - 0,04 \textrm{U}_{n}.$$
 
\noindent \textbf{Partie A}\\
 
\noindent $\triangleright~$\textbf{1)} En fonction de U$_{0}$, 
calculer U$_{1},~ \textrm{U}_{2}$ et U$_{3}$.\\
 
\noindent $\triangleright~$\textbf{2)} Démontrer que cette suite est 
une suite géométrique de premier terme U$_{0}$ et de raison $q$ que l'on 
déterminera.
\\ 
 
\noindent $\triangleright~$\textbf{3)} Quel est son sens de variation ? \\
 
\noindent $\triangleright~$\textbf{4)} Exprimer U$_{n}$ en fonction 
de U$_{0}$ et de $n$.\\
 
\noindent \textbf{Partie B}\\
 
\noindent Le 1$^{\textrm{er}}$ janvier 1997, la population d'une 
commune rurale était de  3000 personnes. On admet que cette population a
diminué de 4 \% par an. \\
 
\noindent $\triangleright~$\textbf{1)} Quelle a été la population de 
cette commune au 1$^{\textrm{er}}$ janvier 1999 ?\\
 
\noindent $\triangleright~$\textbf{2)} Quelle sera la population de 
cette commune au 1$^{\textrm{er}}$ janvier 2000 ?\\
 
\noindent $\triangleright~$\textbf{3)} À partir de quelle année la 
population chutera-t-elle à moins de 2 000 personnes ?\\
 
\noindent \textbf{Exercice 1}\hspace{2cm}5 points\\
\textbf{Commun à tous les candidats}\\
 
\noindent Le tableau suivant donne la moyenne $y$ des maximums de 
tension artérielle en fonction de l'âge $x$ d'une population donnée. 
$$\begin{array}{| c | c | c | c | c | c | c |}\hline
\textrm{\^Age}~ x&	~~36~~&~~	42~~&~~	48~~&~~	54~~&~~	60~~ &~~ 66~~\\ 
\hline
\textrm{Tension}~ y&	12&	13,5&	12,6&	14,3&	15,4&	15\\ \hline
\end{array}$$
 
\noindent $\triangleright~$\textbf{1)} Représenter graphiquement le 
nuage de points $M(x~;~ y)$ dans un repère orthogonal. On prendra pour unités 
graphiques 0,5 cm pour 1 an en abscisse et 3 cm en ordonnée pour l'unité de
tension artérielle, l'origine correspond au point 1 de coordonnées (30 ; 10).\\
 
\noindent $\triangleright~$\textbf{2)} Dans cette partie, vous 
pourrez utiliser votre calculatrice.\\
\textbf{a)} Calculer à $10^{-2}$ près le c¦fficient de corrélation 
entre $x$  et $y$. On admet qu'un ajustement par la méthode des moindres 
carrés est justifié.\\ 
\textbf{b)} Déterminer l'équation de la droite de régression de $y$ 
en $x$ et la représenter (les c¦fficients seront donnés à 0,001 près).\\
\textbf{c)} Une personne de 70 ans a une tension de 16,1. Quelle 
serait sa tension théorique en utilisant la droite de régression ? Comparer
avec la tension réelle.\\
\textbf{d)} Compléter le tableau de l'annexe  en utilisant les 
valeurs de « $a$ » et de « $b$ » obtenues pour la droite de régression.\\
Calculer la somme des « carrés » de la dernière colonne, associée à 
cet ajustement (calcul de la somme des résidus associés à cet ajustement).\\
\textbf{Annexe :}\\ 
À rendre avec la copie (après l'avoir complétée)
\begin{center}
TABLEAU
$ a = \cdots \cdots		b = \cdots \cdots$ \end{center}
$$\begin{array}{| c| c| c| c| c|}\hline
x_{i}&	y_{i}&	ax_{i} + b&	y_{i} - (ax_{i} + b)&	\left[y_{i} - (ax_{i} 
+ b)\right]^2\\ \hline
36 &	12 & & & \\ \hline
42 &	13,5 & & & \\ \hline
48 &	12,6 & & & \\ \hline
54 &	14,3 & & & \\ \hline
60 &	15,4 & & & \\ \hline
66 &    15 & & & \\ \hline
\end{array}$$
 
Somme des « carrés » de la dernière colonne : $\cdots \cdots$.\\
 
\noindent  \textbf{PROBLÈME}\hspace{2cm}	11 points\\
 
\textit{Le but du problème est l'étude d'une fonction et le calcul
d'une aire liée à cette fonction.}\\ 
 
\noindent \textbf{Partie A}\\
 
La courbe ($\Gamma$) ci-jointe (annexe 1) est la représentation 
graphique dans un repère orthonormal d'une fonction $g$ définie et dérivable
sur ]0, +~$\infty$[.\\ 
Les points A$\left( 1 ~;~ \cfrac{3}{2}\right)$ et B $\left(\textrm{e}~ ;~
\cfrac{\textrm{e}^2}{2}\right)$	appartiennent à la courbe ($\Gamma$) et la
tangente en A à ($\Gamma$) est parallèle à l'axe des abscisses.\\
 
\noindent $\triangleright~$\textbf{1)} Déterminer $g(1)~ ;~ g(\textrm{e})$ 
et $g'( 1)$.\\ 
 
\noindent $\triangleright~$\textbf{2)} Déterminer les réels $a$ et 
$b$, sachant que la fonction $g$ est définie sur ]0 ; +~$\infty$[ par une
expression de la forme :
$$g(x) = \cfrac{x^2}{2} + a + b \ln x.$$
 
\noindent $\triangleright~$\textbf{3)} Sachant que $g (x) = \cfrac{x^2}{2} + 
1 - \ln x$, retrouver au moyen d'un calcul, le sens de variation de 
$g$.
(Le calcul des limites n'est pas demandé.)\\
En utilisant ce dernier résultat, étudier le signe de $g$ sur ]0 ; +~$\infty$[.
\\
 
\noindent \textbf{Partie B}\\
 
On considère la fonction $f$ définie sur ]0 ; +~$\infty$[ par $f(x) = 
\cfrac{\ln x}{x} + \cfrac{x}{2}.$\\
 
\noindent $\triangleright~$\textbf{1)} Calculer les limites de $f$ en 
0 et en +~$\infty$.\\
(On admet le résultat suivant : limite en +~$\infty$ de $\cfrac{\ln 
x}{x} = 0$.)\\ 
 
\noindent $\triangleright~$\textbf{2)} Calculer la dérivée $f'$ de 
$f$.\\
Vérifier que $f'(x) = \cfrac{g(x)}{x}$ pour tout réel positif $x$.\\
En déduire les variations de $f$.\\
 
\noindent $\triangleright~$\textbf{3)} Montrer que la représentation 
graphique ($\mathcal{C}$) de $f$ dans un repère orthonormal admet deux 
asymptotes que l'on précisera.\\
La courbe ($\mathcal{C}$) de $f$ est donnée en annexe dans un repère 
orthonormal (0 ;~$\overrightarrow{\imath} ,~\overrightarrow{\jmath}$), unité
2 cm sur chaque axe.\\ 
 
\noindent $\triangleright~$\textbf{4)} On admet l'existence d'un réel 
$\alpha$ unique, appartenant à $\left[\cfrac{1}{2}~;~1\right]$
tel que $f(\alpha) = 0$. Que représente $\alpha$ pour la courbe 
($\mathcal{C}$) ? Placer sur la courbe ($\mathcal{C}$) le point I d'abscisse 
$\alpha$. Montrer que $\ln \alpha = -~\cfrac{\alpha^2}{2}$. En déduire que
$f'(\alpha) = \cfrac{1 + \alpha^2}{\alpha^2}$.\\
 
\noindent \textbf{Partie C}\\
 
\noindent $\triangleright~$\textbf{1)} Calculer  la dérivée de la 
fonction $h$ définie sur ]0 ; +~$\infty$[ par $h(x) = (\ln x)^2$ .\\
 
\noindent $\triangleright~$\textbf{2)} En déduire le calcul de J = 
$\int\limits_{1}^{t} \left(\cfrac{\ln x}{x}\right)\: \textrm{d}x.$
 
\noindent $\triangleright~$\textbf{3)} Hachurer sur le graphique 
donné en annexe le domaine plan limité par ($\mathcal{C}$), l'axe des
abscisses, et les  droites d'équations $x = 1$ et $x = \textrm{e}$.\\ 
Déterminer l'aire, en cm$^2$ , de ce domaine.\\
\newpage
\begin{center}
    \textit{Annexe} 2\\
    \textit{À rendre avec la copie (après l'avoir complétée)\\
    Courbe} $(\Gamma$)\\
    \vspace{1cm}
    \begin{pspicture}(4,5.5)
	\psline{->}(0,-0.5)(0,5.5) \psline{->}(-1,0)(4,0)
	\psline[linestyle=dashed](1,0)(1,1.5) 
	\psline[linestyle=dashed](2.718,0)(2.718,3.695) 
	\psline[linestyle=dashed](0,1.5)(1,1.5)
	\rput(1,1.65){A} \rput(2.6,3.8){B} \rput(-0.3,-0.3){O}
	\rput(4,-0.3){$x$}	\rput(-0.3,5.2){$y$}
	\psplot[plotpoints=1000]{0.2}{3}{x 2 exp 2 div 1 add x ln sub}
	\end{pspicture}
	\vspace{3cm}\\
 
\textit{Courbe} ($\mathcal{C}$)\\
 
	 \begin{pspicture}(6,5.5)
	     \psline{->}(-1,2)(6,2)
	     \psline{->}(0,0)(0,5.5)
	     \rput(-0.3,1.7){O} \rput(6,1.8){$x$} \rput(-0.3,5.2){$y$}
	     \rput(2.2,1.8){$\mathcal{C}$} 
\psplot[plotpoints=1000]{0.4}{5.5}{x  2 div  x ln  x div  add}
\end{pspicture}
\end{center}
\footnote{\scalebox{1 -1}{Antilles juin   1999}}\\
\newpage
 
%% Sujet 4    %%%%
%% Centres étrangers Groupe 1 juin 1999 %%
 
\noindent  \doublebox{TES   
\hspace{2.9cm}\Large{\textbf{Baccalauréat juin 1999}} 
\hspace{2.9cm}\normalsize{}}\\
 
\noindent \textbf{Exercice 1}\hspace{2cm}4 points\\
 
\textit{Aucun détail des calculs effectués à la calculatrice n'est
exigé dans cet exercice.}\\ 
Le tableau ci-dessous donne l'évolution du chiffres d'affaires 
réalisé à l'exportation par une entreprise. 
$$\begin{array}{*{10}{|c|}}\hline
\textrm{Année}& 1990&	1991&	1992&	1993&	1994&	1995&	1996&	1997&	
1998\\ \hline
x_{1}&	0&	1&	2&	3&	4&	5&	6&	7&	8\\ \hline
y_{i}&	100&  101&	107&	122&	127&	139&	136&	157&	165\\ \hline
\end{array}$$
$x_{1}$ désigne le rang de l'année,\\
$y_{i}$ désigne l'indice du chiffre d'affaires à l'exportation 
rapporté à la base 100
en 1990.\\
 
\noindent~$\triangleright~$\textbf{1) a)} Représenter le nuage de points 
$M_{i}(x_{i~}~;~ y_{i})$ associé
à la série double dans un repère orthogonal. On prendra :\\ 
$\bullet$  pour origine le point $M_{0}(0~;~ 100)$,\\ 
$\bullet$  pour unités : 1,5 cm sur l'axe des abscisses,\\ 
\hspace{4,5cm}2 cm pour 10 points d'indice sur l'axe des ordonnées.\\
\textbf{b)} Calculer les coordonnées du point moyen G associé à cette 
série statistique et placer ce point sur le graphique. (On donnera la valeur
décimale arrondie au  dixième de l'ordonnée de G.)\\
 
\noindent~$\triangleright~$\textbf{2)} Déterminer la valeur décimale 
arrondie au centième du c¦fficient de corrélation linéaire de la série 
double. Ce résultat permet-il d'envisager un ajustement affine ? Pourquoi ?\\ 
 
\noindent~$\triangleright~$\textbf{3)} Soit $\mathcal{D}$, la droite 
d'ajustement de $y$ en $x$ obtenue par la méthode des moindres carrés.\\ 
\textbf{a)} Donner la valeur décimale arrondie au dixième du 
c¦fficient directeur de la droite $\mathcal{D}$.\\ 
\textbf{b)}  En utilisant les coordonnées du point moyen G, donner 
une équation de la droite $\mathcal{D}$.\\ 
Tracer cette droite sur le graphique précédent.\\
 
\noindent $\triangleright~$\textbf{4)} En supposant que l'évolution 
du chiffre d'affaires se poursuive de la même façon au cours des années
suivantes, estimer l'indice du chiffre d'affaires de cette entreprise en l'an
2001 (on en donnera la valeur arrondie à l'unité).\\ 
 
\noindent \textbf{Exercice 2}\hspace{2cm}5 points\\
\textbf{(obligatoire)}\\
 
Une étude statistique indique que 95 \% des téléviseurs fabriqués par 
une entreprise sont en état de fonctionnement. On fait subir à chaque 
appareil un test de contrôle.\\
On constate que :\\
$\bullet$ quand un appareil est en état de fonctionnement, il est 
accepté dans 96 \% des cas à l'issue du test ;\\
$\bullet$ quand un appareil n'est pas en état de fonctionnement, il 
est néanmoins accepté dans 8 \% des cas à l'issue du test.\\
On choisit au hasard un téléviseur fabriqué par l'entreprise.\\
On définit les événements suivants :\\
F : « le téléviseur est en état de fonctionnement » ;\\
T : « le téléviseur est accepté à l'issue du test » ;\\
T : « le téléviseur est refusé à l'issue du test ».\\
Ainsi :\\
$\bullet$ la probabilité de l'événement F, notée P(F) est 0,95 ;\\ 
$\bullet$ la probabilité P(T/F) qu'un téléviseur soit accepté à 
l'issue du test sachant qu'il est en état de fonctionnement est 0,96.\\ 
 
\noindent $\triangleright~$\textbf{1)} Calculer la probabilité que le 
téléviseur ne soit pas en état de fonctionnement.\\ 
 
\noindent $\triangleright~$\textbf{2) a)} Calculer la probabilité 
qu'un téléviseur soit refusé à l'issue du test sachant qu'il est en
état de fonctionnement.\\ 
\textbf{b)} Calculer la probabilité que le téléviseur soit refusé à 
l'issue du test et qu'il soit en état de fonctionnement.\\
\textbf{c)} Calculer la probabilité que le téléviseur soit refusé à 
l'issue du test et qu'il ne soit pas en état de fonctionnement.\\
 
\noindent $\triangleright~$\textbf{3)} En déduire la probabilité pour 
que le téléviseur soit refusé à l'issue du test.\\ 
 
\noindent $\triangleright~$\textbf{4)} Quelle est la probabilité pour 
qu'un téléviseur soit en état de fonctionnement sachant qu'il est refusé
à l'issue du test ? (On donnera la valeur décimale arrondie au millième du
résultat.) \\
 
 
\noindent \textbf{Exercice 2}\hspace{2cm}5 points\\
\textbf{(spécialité)}\\
 
Le salaire annuel d'un technicien s'élevait pour l'année 1998 
à 90\:000 F.\\
Chaque année son employeur décide de l'augmenter de 2 \% et de lui 
allouer en plus 5\:000 F.\\
On désigne par S$_{0}$ le salaire du technicien pour l'année 1998. 
Pour tout entier naturel $n$, on désigne par S$_{n}$ son salaire pour l'année
(1998 + $n$).\\
Par exemple : S$_{2}$ est le salaire du technicien pour l'année 
2000.\\
 
\noindent $\triangleright~$\textbf{1)} Calculer S$_{1}$ et S$_{2}$.\\ 
 
\noindent $\triangleright~$\textbf{2)} Pour tout entier naturel $n$, 
exprimer S$_{n+1}$ en fonction de S$_{n} $.\\
 
\noindent $\triangleright~$\textbf{3)} On définit la suite (U$_{n}$) 
par U$_{n}$ = S$_{n}$ + 250\:000 pour tout entier naturel.\\
\textbf{a)} Calculer U$_{0}$.\\
\textbf{b)} Montrer que la suite (U$_{n}$) est une suite géométrique 
de raison 1,02.\\
\textbf{c)} Exprimer U$_{n}$ en fonction de $n$.\\ 
 
\noindent $\triangleright~$\textbf{4) a)} Exprimer S$_{n}$ en 
fonction de $n$.\\
\textbf{b)} En déduire le salaire prévu pour l'année 2005.\\ 
 
\noindent $\triangleright~$\textbf{5)} À partir de quelle année le 
salaire de ce technicien aura-t-il doublé ?\\ 
 
\noindent \textbf{Problème}\hspace{2cm}11 points\\
 
\noindent L'objet de ce problème est l'étude d'une fonction et le 
tracé de sa représentation graphique (partie B) s'appuyant sur l'étude
d'une fonction auxiliaire (partie A).\\
On calculera enfin une aire (partie C). On prendra soin de faire figurer sur la
copie les calculs intermédiaires conduisant aux résultats.\\ 
 
\noindent \textbf{Partie A}\\
 
\noindent~$\triangleright~$\textbf{1)} Soient $a,~ b$ et $c$ des nombres réels. 
On définit une fonction $g$ sur $\R$ par $g(x) = (ax + b)e^{-x} + c$. 
On note $g'$ la fonction dérivée de $g$.\\ 
\textbf{a)} Calculer $g'(x)$.\\
\textbf{b)} Le tableau de variation de $g$ est le suivant :\\ 
\begin{center}
\begin{pspicture}
\psset{xunit=10mm,yunit=10mm}
    \begin{pspicture}(-0.5,0.4)(8.5,-3)
\psline(-0.5,0)(8.5,0) \psline(-0.5,-1)(8.5,-1)  \psline(0,0.4)(0,-3)
\psline{->}(1,-2.5)(5,-1.2) \psline{->}(6.3,-1.2)(8.2,-2.2) 
\psline(6,0)(6,-1) 
\rput(-0.5,0.2){$x$}  \rput(-0.5,-0.5){$g'(x)$} \rput(-0.5,-2){$g(x)$}
\rput(0.4,0.2){$-~\infty$} \rput(2.5,0.2){0} \rput(4,0.2){1} 
\rput(6,0.2){2}
\rput(8.2,0.2){$+~\infty$}
\rput(3.5,-0.5){+} \rput(6,-0.5){0} \rput(7,-0.5){-}
\rput(0.4,-2.6){$-~\infty$} \rput(2.5,-2){1} \rput(4,-1.6){2}
\rput(5.6,-1.3){$e^{-2} + 2$} \rput(8.3,-2.1){2}
\end{pspicture}
\end{center}
En utilisant les données numériques de ce tableau, établir que $a = 
1,~ b = -~1$ et $c = 2$.\\
Ainsi, pour la suite du problème : $g(x) = (x - 1 )e^{-~x} + 2$.\\
 
\noindent~$\triangleright~$\textbf{2) a)} Montrer que l'équation $g (x) = 0$ 
admet une solution unique dans l'intervalle $[ -~1~ ;~ 0]$ . On note 
$\alpha$ cette solution.\\
\textbf{b)} Déterminer à l'aide de la calculatrice la valeur 
décimale arrondie au dixième de $\alpha$.\\
 
\noindent~$\triangleright~$  \textbf{3)} Étudier le signe de $g(x)$ pour $x$
 appartenant à $\R$.\\
 
\noindent \textbf{Partie B}\\
 
Soit $f$ la fonction définie sur $\R$ par $f(x) = 2 x + 1 - xe^{- x}$.\\
 
\noindent~$\triangleright~$\textbf{1) a)} Déterminer la limite de $f$ en 
$+~\infty$ (on admettra que $\lim\limits_{x \to +~\infty} 
\cfrac{e^x}{x} = +~\infty$).\\
\textbf{b)} Déterminer la limite de $f$ en $-~\infty$ (on pourra 
mettre $x$ en facteur dans l'expression de $f(x)$).\\
 
\noindent~$\triangleright~$\textbf{2)  a)} Soit $f'$ la fonction dérivée de 
$f$. Montrer que $f'(x) = g (x)$.\\
\textbf{b)} Dresser, en le justifiant, le tableau de variation de $f$ 
sur $\R$.\\
 
\noindent~$\triangleright~$\textbf{3)} Dans le plan muni d'un repère 
orthonormal $(0~ ;~ \overrightarrow{\imath}~,~\overrightarrow{\jmath})$, on 
appelle $(\mathcal{C})$ la représentation graphique de $f$ et $(\mathcal{D})$ 
la droite d'équation $y = 2x + 1$.\\
\textbf{a)} Déterminer $\lim\limits_{x \to +~\infty} [f(x) - (2x + 1)]$.\\
\textbf{b)} Donner une interprétation graphique de ce résultat.\\
\textbf{c)} Étudier la position de $(\mathcal{C})$ par rapport à 
$(\mathcal{D})$.\\
\textbf{d)} Tracer $(\mathcal{D})$ et $(\mathcal{C})$ dans le plan 
muni du repère orthonormal $(0~ ;~ \overrightarrow{\imath}~,~\overrightarrow
{\jmath})$. On prendra pour unité graphique 2 cm.
 
\noindent \textbf{Partie C}\\
 
Soient $H$ la fonction définie sur $\R$ par $H(x) = -~e^{- x}(1 +  
x)$ et $h$ la fonction définie sur $\R$ par $h(x) = xe^{ - x}$.\\
 
\noindent~$\triangleright~$\textbf{1)} Montrer que la fonction $H$ est une 
primitive sur $\R$ de la fonction $h$.\\
 
\noindent~$\triangleright~$\textbf{2)} Hachurer sur le graphique précédent le 
domaine limité par la courbe $(\mathcal{C})$, la droite $(\mathcal{D})$ et 
les droites d'équations $x = 0$ et $x = 1$.\\
 
\noindent~$\triangleright~$\textbf{3)} Calculer l'aire $S$ en cm$^2$ du domaine
hachuré.\\ 
\footnote{\scalebox{1 -1}{Centres étrangers 1999}}\\
\newpage
%% Sujet 5 Polynésie juin 1999 %%
\noindent  \doublebox{TES   
\hspace{2.9cm}\Large{\textbf{Baccalauréat juin 1999}} 
\hspace{2.9cm}\normalsize{}}\\
 
\noindent \textbf{Exercice 1}\hspace{2cm}4 points\\
\textbf{Commun à tous les candidats}\\
 
On considère une fonction $f$ définie et dérivable sur l'intervalle
[1 ; 6]. Sa courbe représentative ($\mathcal{C}$) dans un repère 
orthogonal est donnée ci-dessous.\\
La courbe ($\mathcal{C}$) passe par les points
A(1 ; 0), B(2 ; 1), D(4 ; 4) et E(6 ; 1).
Les tangentes à la courbe aux points A et D sont parallèles à l'axe 
des abscisses.\\
La tangente à la courbe au point E passe par le point F(5 ;  5).
\begin{center}
\begin{pspicture}(6,5)
    \psgrid[subgriddiv=1]
    \rput(0.8,0.2){A} \rput(1.8,1.2){B} \rput(2.5,2.7){$\mathcal{C}$}
    \rput(4.2,4.2){D} \rput(6.2,0.9){E} 
    \psline{->}(0,0)(1,0) \psline{->}(0,0)(0,1)
    \pscurve(1,0)(1.5,0.38)(2,1)(3,3)(4,4)(5,3.3)(6,1)
    \psline{<->}(3.5,4)(4.5,4) \psline(6,1)(5,5)  
\psline{->}(6,1)(5.5,3)
    \psline{<->}(0,0)(2,0)
    \end{pspicture}
    \end{center}
    \vspace{1cm}
 
\noindent \textbf{Partie I}\\
Par lecture graphique, résoudre l'équation $f(x) = 0$ et donner le 
signe de $f(x)$ sur l'intervalle [ 1 ; 6].\\
 
\noindent \textbf{Partie II}\\
On désigne par $g$ la fonction définie sur l'intervalle ] 1 ; 6 ] par
$g(x) = \cfrac{1}{f(x)}$ et ($\Gamma$) sa courbe représentative dans 
un repère orthonormal d'unité graphique 2 cm.\\
 
\noindent $\triangleright~$\textbf{1) a)} Calculer $g(2),~ g(4)$ et 
$g(6)$.\\
\textbf{b)} Déterminer la limite de $g(x)$ quand $x$ tend vers 1.\\
Que peut-on en déduire pour la courbe ($\Gamma$) ?\\
\textbf{c)} Dresser le tableau de variation de la fonction $g$ sur 
l'intervalle ] 1 ; 6 ] en donnant les justifications nécessaires.\\ 
\textbf{d)} Déterminer $f'(4)$ ; en déduire $g'(4)$.\\
 
\noindent $\triangleright~$\textbf{2)} Tracer la courbe ($\Gamma$) 
ainsi que son asymptote et la tangente au point d'abscisse 4.\\ 
 
\noindent \textbf{Exercice 2}\hspace{2cm} 4 points\\
\textbf{Candidats n'ayant pas suivi l'enseignement de spécialité}\\
 
Le tableau suivant donne pour les années indiquées, le nombre de 
demandes d'emploi en fin d'année dans une région.\\
\begin{center}
\begin{tabular}{|l |r @{\:}l |r@{\:} l|}\hline
     & \multicolumn{2}{|c |}{1996} & \multicolumn{2}{| c |}{1997}\\ 
     \hline
     Total & ~~~85 &079~~~ & ~~~85 & 240~~~\\ \hline
Moins de 25 ans &22 &238 & 20&276\\ 
De 25 ans à 39 ans & 54 & 719 & 55 & 994\\
50 ans et plus& 8 & 122 & 8 & 970\\ \hline
Hommes & 39 & 998 & 39 & 766\\
Moins de 25 ans & 10 & 176 & 9 & 170\\
De 25 ans à 39 ans& 25 & 528 & 25 & 853\\
50 ans et plus& 4 &284 & 4 & 743\\ \hline 
Femmes & 45 & 091 & 45 & 474\\
Moins de 25 ans & 12 & 062 & 11 & 106\\
De 25 ans à 39 ans & 29 & 191 & 30& 141\\
50 ans et plus & 3 & 838 & 4 & 227\\ \hline
\end{tabular}\\
\tiny{Source: ANPE-INSEE Poitou-Charentes.}\normalsize{}\\
\end{center}
 
\noindent \textit{Les résultats des calculs seront donnés sous forme 
approchée à $10^{- 2}$ près par défaut.}\\
 
\noindent$\triangleright$~\textbf{1) a)} Déterminer le pourcentage d'évolution 
du total des demandes d'emploi entre 1996 et 1997.\\
\textbf{b)} Le nombre de demandes d'emploi est en baisse pour une 
tranche d'âge seulement.\\ 
Calculer le pourcentage d'évolution des demandes d'emploi des hommes 
pour cette tranche d'âge.\\
 
\noindent$\triangleright$~\textbf{2)} En 1996, une entreprise est 
subventionnée pour employer une personne de moins de 25 ans. 
Elle choisit une personne au hasard parmi les demandeurs d'emploi 
concernés. 
Tous les choix sont équiprobables.\\
Quelle est la probabilité que la personne embauchée soit une femme ?\\
 
\noindent$\triangleright$~\textbf{3)} L'entreprise désire créer un emploi en 
1998 et choisit au hasard une personne dans les demandeurs d'emploi 
de 1997. Tous les choix sont équiprobables. 
Calculer la probabilité $p$ que la personne embauchée soit un homme.\\ 
Vérifier que 0,46 est une valeur approchée par défaut à $10^{-2}$ près de 
$p$.\\
 
\noindent$\triangleright$~\textbf{4)} Dans cette question, on 
prendra $p$ égal à 0,46.\\
L' entreprise choisit trois demandeurs d'emploi de 1997.\\
\textit{Les choix sont indépendants et on assimilera ce choix à un tirage avec
remise.}\\
\textbf{a)} Quelle est la probabilité qu'elle choisisse trois hommes ?\\
\textbf{b)} Quelle est la probabilité qu'elle choisisse un homme et un 
seul ?\\
\textit{On pourra utiliser un arbre pondéré.}\\
 
\noindent \textbf{Exercice 2}\hspace{2cm}4 points\\
\textbf{Candidats ayant suivi l'enseignement de spécialité}\\
 
Pour financer ses études, une étudiante fait du démarchage par
téléphone pour vendre un produit qui lui rapporte 20 francs. Elle ne 
peut vendre qu'un produit par appel.\\ 
Lorsqu'elle compose un numéro de téléphone, trois possibilités se 
présentent :\\ 
$\bullet$~ l'événement $A$ « Personne ne répond » de probabilité 
$p(A)$ égale à 0,3~;\\ 
$\bullet$~ l'événement $B$ « Le répondeur téléphonique diffuse un 
message » avec une probabilité $p(B)$ égale à 0,1~ ;\\ 
$\bullet$~l'événement $C$ « Un correspondant répond » de probabilité 
$p(C)$ égale à 0,6.\\
 
\noindent$\triangleright$~\textbf{1)} La probabilité que l'étudiante vende son 
produit sachant qu'un correspondant répond à son appel est égale à 0,4.\\
Les probabilités qu'elle vende son produit dans les autres cas sont 
nulles.\\
 Vérifier que la probabilité que l'étudiante réalise une vente lors 
d'un appel téléphonique fait au hasard est égale à 0,24.\\
 
\noindent$\triangleright$~\textbf{2)} Lorsque personne ne répond à son appel 
téléphonique, l'étudiante débourse 0 franc.\\ 
Lorsqu'un répondeur téléphonique diffuse un message, l'étudiante 
débourse 1 franc.\\ 
Lorsqu'un correspondant répond, l'appel coûte 1 franc et dans ce cas \\
\hspace*{0,9cm}-- si l'étudiante vend son produit, qui lui 
rapporte 20 francs, elle aura donc fait un gain de + 19 francs,\\ 
\hspace*{0,9cm}-- si elle ne vend pas son produit, elle aura 
perdu 1 franc.\\
On considère la variable aléatoire $X$ correspondant au gain 
algébrique possible lors d'un appel téléphonique de l'étudiante.\\
\textbf{a)} Démontrer que la probabilité que le gain algébrique soit 
égal à -1 est 0,46.\\
\textbf{b)} Déterminer la loi de probabilité de la variable aléatoire 
$X$.\\
\textbf{c)} Calculer l'espérance mathématique de $X$.\\
 
\noindent$\triangleright$~\textbf{3)} On suppose que l'étudiante compose 
successivement de manière indépendante cinq numéros de téléphone au
hasard.\\
Déterminer la probabilité qu'elle réalise exactement trois ventes.\\
 
\noindent \textbf{PROBLÈME }\hspace{2cm} 12 points\\
 
Le plan est rapporté à un repère orthonormal. On prendra pour unité
graphique 2 cm.\\
On considère les fonctions $f$ et $g$ définies sur l'intervalle 
$[0~;~+~\infty[$ par
$$f(x) = (-~x + 4)e^{x- 1}~ \textrm{et}~ g(x) = \ln\left(\cfrac{x + 6}
{2x + 2}\right)$$
Dans le repère choisi, on appelle $(\mathcal{C})$ la courbe représentative de 
$f$ et $(\Gamma)$ la courbe représentative de $g$.\\ 
 
\noindent \textbf{Partie A}\\
 
\noindent $\triangleright~$\textbf{1)} Déterminer la limite de $f(x)$ quand $x$ 
tend vers $+~\infty$.\\
 
\noindent$\triangleright~$\textbf{2)} Vérifier que la fonction dérivée de $f$ 
est définie pour tout $x$ positif par $f'(x) = (- x + 3)e^{x - 1}$ .\\
 
\noindent $\triangleright~$\textbf{3)} Étudier le sens de variation de la 
fonction $f$ et dresser son tableau de variation. On précisera $f(0),~ 
f'(0),~f(3),~ f'(3).$\\
 
\noindent $\triangleright~$\textbf{4)} Tracer la courbe $(\mathcal{C})$.\\
 
\noindent $\triangleright~$\textbf{5)} Déterminer les réels $a$ et $b$ tels
que la fonction $F$ définie sur l'intervalle $[0~;~+~\infty[$ par $F(x) = 
(ax + b)e^{x - 1}$ soit une primitive de la fonction $f$.\\ 
 
\noindent \textbf{Partie B}\\
On considère la fonction $u$ définie sur l'intervalle 
$[0~;~+~\infty[$ par
$$u(x) = \cfrac{x + 6}{2x + 2}$$
 
\noindent~$\triangleright$\textbf{1)} Vérifier que, pour tout $x$ positif 
 $u(x)$ est strictement positif.\\
 
\noindent~$\triangleright$\textbf{2) a)} Déterminer la limite de $u(x)$ quand 
$x$ tend vers $ +~\infty$.\\
\textbf{b)} Étudier le sens de variation de $u$.\\ 
Dresser le tableau de variation de $u$ et retrouver le résultat de la 
question \textbf{1)} de la partie \textbf{B}.\\
 
\noindent~$\triangleright$\textbf{3)} En utilisant les résultats précédents, 
déterminer le sens de variation de la fonction $g$ et démontrer que 
la courbe $(\Gamma)$ admet une asymptote $(D)$ au voisinage de $+~\infty$ dont 
on donnera une  équation.\\
 
\noindent~$\triangleright$\textbf{4)} Tracer la courbe $(\Gamma)$ et la droite 
$(D)$ sur le même graphique que celui de la partie A.\\
 
\noindent~$\triangleright$\textbf{5)} Soit $G$ la fonction définie sur 
l'intervalle  $[0~;~+~\infty[$  par
$$G(x) = (x+ 6)\ln(x + 6) - (x + 1)\ln(2x + 2).$$
Démontrer que $G$ est une primitive de la fonction $g$ sur l'intervalle 
$[0~;~+~\infty[$.\\
 
\noindent \textbf{Partie C}\\
 
\noindent~$\triangleright$\textbf{1)} Résoudre, à l'aide des représentations 
graphiques faites, l'inéquation $g(x) ¾ f (x) .$\\
 
\noindent~$\triangleright$\textbf{2)} Calculer l'aire $\mathcal{A}$ en cm$^2$ du
domaine du plan constitué des points $M(x~;~ y)$ tels que : 
$$2 ¾ x ¾ 3~ \textrm{et}~ g(x) ¾ y ¾ f(x).$$ 
Donner l'arrondi de $\mathcal{A}$ à l'unité près.\\
\footnote{\scalebox{1 -1}{Polynésie 1999}}\\
\newpage
 
%%%%       Sujet 6           %%%%
%%%% La Réunion juillet 1999 %%%%
 
\noindent  \doublebox{TES   
\hspace{2.9cm}\Large{\textbf{Baccalauréat juin 1999}} 
\hspace{2.9cm}\normalsize{}}\\
 
\noindent \textbf{Exercice 1}\hspace{2cm}4 points\\
 
Une entreprise est équipée d'ordinateurs de trois modèles différents.\\
30 \% sont de marque (M$_1$), 50 \% sont de marque (M$_2$) et 20 \% de marque 
(M$_3$).\\
On choisit un appareil au hasard. Tous les choix sont équiprobables.
Pour $i$ égal à 1, 2 ou 3, on appelle M$_i$ l'événement : « l'appareil choisi est
 de marque (M$_i$) ».\\
On note $p$(M$_i$) la probabilité de l'événement M$_i$.\\
on a donc $p$(M$_{1}$) = 0,3 ; $p$(M$_{2}$) = 0,5 et $p$(M$_{3}$) = 0,2.\\
On note T l'événement : « l'appareil choisi tombe en panne » et $p$(T) la 
probabilité de cet événement.\\
On suppose que si un appareil tombe en panne, il est réparé et qu'il
fonctionne alors correctement.\\
La probabilité $p_1$(T) qu'un appareil de marque (M$_1$) tombe en panne 
est $\cfrac{1}{30}$.\\
La probabilité $p_2$(T) qu'un appareil de marque (M$_2$) tombe en panne est 
$\cfrac{1}{20}$.\\
La probabilité $p_3$(T) qu'un appareil de marque (M$_3$) tombe en panne est 
$\cfrac{1}{40}$.\\
 
\noindent $\triangleright~$\textbf{1) a)} Traduire toutes les données sur un
 arbre pondéré.\\
\textbf{b)} Calculer la probabilité que l'appareil choisi soit de marque 
(M$_2$) et qu'il tombe en panne.\\ 
\textbf{c)} Vérifier que la probabilité qu'un ordinateur tombe en panne est
égale à 0,04.\\
\textbf{d)} Quelle est la probabilité que l'appareil soit de marque (M$_2$)
 sachant qu'il est tombé en panne ?\\
 
\noindent $\triangleright~$\textbf{2)} \textit{Dans cette question, on donnera le
 résultat à} 0, 1 \textit{près}.\\
Un service de l'entreprise possède quatre ordinateurs.\\
On suppose que les pannes éventuelles de ces ordinateurs sont indépendantes
deux à deux.\\
Quelle est la probabilité qu'aucun des quatre ordinateurs ne tombe en panne ?\\
 
\noindent \textbf{Exercice 2}\hspace{2cm}5 points\\
\textbf{(obligatoire)}\\ 
 
Dans cet exercice aucun détail des calculs statistiques effectués à la
 calculatrice n'est demandé.\\ 
Lors d'une période de sécheresse, un agriculteur relève la quantité totale 
(en m$^3$) utilisée par son exploitation depuis le premier jour et donne le
 résultat suivant :
\begin{center}
\begin{tabular}{| l *5{| c}| l}\hline
\textbf{Nombre de jours écoulés :} $x_i$ &  1 & 3 & 5 & 8 & 10\\ \hline
\textbf{Volume utilisé (en m}$^3)~: y_i$ &	 2,25 & 4,3 &	8 &  17,5 & 27\\ \hline
\end{tabular}\\
\end{center}
Le plan est muni d'un repère orthogonal. On prendra pour unité graphique sur
 l'axe des abscisses 1 cm pour un jour et sur l'axe des ordonnées 0,5 cm pour un
 mètre-cube.\\ 
 
\noindent~$\triangleright~$\textbf{1)} Représenter alors la série statistique
 $(x_i~;~ y_i)$.\\
 
\noindent$\triangleright~$\textbf{2) a)} Donner le c¦fficient de corrélation
 linéaire de la série $(x_i~;~ y_i)$ en arrondissant le résultat lu sur la
 calculatrice à $10^{-3}$ près.\\
\textbf{b)} Donner l'équation de $\Delta$ droite de régression de $y$ en $x$
 obtenue par la méthode des moindres carrés sous la forme $y = \alpha x + 
 \beta$$\alpha$ et $\beta$ sont les arrondis à $10^{-2}$ près des valeurs
 lues sur la calculatrice.\\
\textbf{c)} Représenter la droite $\Delta$ sur le graphique.\\
 
\noindent$\triangleright~$\textbf{3)} Le nuage de points permet d'envisager un
 ajustement par la parabole $\mathcal{P}$ qui passe par les points A(1 ; 
 2,25) ; B(10 ; 27) et qui a pour équation $y = ax^2 + b$$a$ et $b$ sont 
 deux nombres réels.\\ 
\textbf{a)} Déterminer $a$ et $b$ et donner l'équation de la parabole 
$\mathcal{P}$.\\
\textbf{b)} Représenter la parabole $\mathcal{P}$ sur le graphique.\\ 
 
\noindent$\triangleright~$\textbf{4)} Dans cette question on compare les deux
 ajustements à l'aide du tableau suivant : 
$$\begin{array}{ *{6}{| c} | p{2,5cm}}\cline{1-6}
x_i &  1 & 	3 &	5 & 	8 & 	10 & \\\cline{1-6}
y_i & ~~2,25~~ &~~4,3~~ & ~~8~~ & ~~17,5~~ &~~ 	27~~ & \\ \hline
\left|y_i - \alpha x^2_i + \beta \right| & 2,54 & 0,91 & 2,71 &  &  &
\multicolumn{1}{c|}{\textrm{Total T}_1 :  }\\ \hline 
\left|y_i - a x^2_i + b \right| & 0 & 0,05 & 0,25 &  &  &
\multicolumn{1}{c|}{\textrm{Total T}_2 : }\\ \hline
\end{array}$$
On ne demande pas de recopier ce tableau.\\ 
Les deux totaux calculés évaluent pour chaque ajustement la somme des
écarts entre les ordonnées des points du nuage et les ordonnées des points
de même abscisse de l'ajustement.\\
Donner les arrondis à $10^{-1}$ près des deux totaux T$_1 $ et T$_2$ calculés
 ci-dessus.\\
(Aucun détail n'est demandé.)\\
En déduire l'ajustement qui parait le mieux adapté.\\
 
\noindent \textbf{Exercice 2}\hspace{2cm}5 points\\
\textbf{(spécialité)}\\
 
Dans cet exercice aucun détail des calculs effectués à la calculatrice n'est
 demandé. Dans une région de 1000 km$^2$, la superficie des terrains
 urbanisés entre 1970 et 1998 est donnée par le tableau suivant : 
\begin{center} \begin{tabular}{*{9}{| c}|}\hline
\textbf{Années} & 	1970&	1974&	1978&	1982&	1986&	1990& 1994 & 1998\\
 \hline
\textbf{Rang :} $x_i$ &	0 &	4 &	8 &	12 & 	16 & 	20 & 24 &28
\\ \hline  
\textbf{Superficie
(en km}$^2$) : $y_i$ & 	80 & 	94 & 	110 & 	129 & 	152 & 	178 & 205 & 236\\
\hline
Y$_i$ & 4,38 & 	4,54 & 	 4,70 &	4,86 &	5,02 &	5,18 & 5,32 & 5,46\\ \hline
\end{tabular}\\ \end{center}
Le nuage de points associé à la série statistique $(x_i~;~ y_i)$ est 
représenté ci-dessous.\\
 
\begin{center} \begin{pspicture}(9,9)
\psgrid[gridlabelcolor=white](0,0)(9,9)
\rput(-0.8,0.5){80} \rput(-0.8,1.5){100} \rput(-0.8,2.5){120} \rput(-0.8,4){150} 
\rput(-0.8,6.5){200} \rput(-0.8,9){250}
\rput(1,-0.5){4}  \rput(2,-0.5){8} \rput(3,-0.5){12} \rput(4,-0.5){16} 
\rput(5,-0.5){20} \rput(6,-0.5){24} \rput(7,-0.5){28} 
\rput(8,-0.5){Années}
\rput(0,0.5){*} \rput(1,1.2){*} \rput(2,2){*} \rput(3,2.45){*} 
\rput(4,4.1){*} \rput(5,5.4){*} \rput(6,6.75){*} \rput(7,8.3){*}
\rput(4.4,4){A} \rput(7.4,8.3){B}
\rput(1,9){Superficie}
\end{pspicture} \end{center}
\vspace{1cm}
 
\noindent \textit{Les estimations de superficie demandées dans l'exercice
seront données en} km$^2$ \textit{et arrondies à l'unité}.\\ 
 
\noindent$\triangleright~$\textbf{1) a)} Donner l'arrondi $r$ à $10^{-2}$ près
 du c¦fficient de corrélation linéaire de la série $(x_i~;~ y_i)$.\\ 
\textbf{b)} Donner l'estimation E$_1$ obtenue par la méthode des moindres
carrés de la superficie des terrains urbanisés en 2010.\\ 
 
\noindent$\triangleright~$\textbf{2)} Au vu de la forme du nuage, on effectue
 un autre ajustement.\\
On calcule $\ln y_i$. On appelle Y$_i$ l'arrondi à $10^{-2}$ près de $\ln 
y_i$. Les valeurs Y$_i$ sont données dans le tableau considéré.\\
\textbf{a)} Donner l'arrondi $r'$ à $10^{-4}$ près du c¦fficient de corrélation
 linéaire de la série $(x_i$~;~ Y$_i)$.\\
\textbf{b)} On prendra Y = $0,039x + 4,39$ pour équation de la droite de
 régression de Y en $x$ obtenue par la méthode des moindres carrés.\\
Calculer la valeur $y$ estimée pour l'année 2010.\\
En déduire une estimation E$_2$ de la superficie des terrains urbanisés en 
2010.\\
 
\noindent$\triangleright~$\textbf{3)} On fait un troisième ajustement du nuage
 de points en utilisant la droite $\mathcal{D}$ passant par les points A(16 ;
 152) et B(28 ; 236).\\
\textbf{a)} Donner une équation de la droite $\mathcal{D}$.\\ 
\textbf{b)} En déduire l'estimation E$_3$ faite avec cet ajustement, de la
 superficie des terrains urbanisés en 2010.\\
 
\noindent \textbf{Problème}\hspace{2cm}11 points
 
\begin{center} \textbf{Partie A} \end{center}
On considère la fonction $g$ définie sur $\R$ par $g(x) = 1 + (- x + 
2)\textrm{e}^{-x}$.\\ 
 
\noindent$\triangleright~$\textbf{1)} Calculer $g'(x)$ ou $g'$ désigne la
 fonction dérivée de $g$ et étudier son signe selon les valeurs de $x$.\\ 
 
\noindent$\triangleright~$\textbf{2)} Étudier le sens de variation de la
 fonction $g$ sur $\R$. Préciser $g(3)$.\\ 
On ne demande pas les limites en + ~$\infty$ et en - ~$\infty$.\\ 
 
 \noindent $\triangleright~$\textbf{3)} En déduire le signe de $g(x)$  sur 
 $\R$.\\ 
 
\begin{center} \textbf{Partie B} \end{center}
On considère la fonction $f$ définie sur $\R$ par $f(x) = x + (x - 
1)\textrm{e}^{-~x}$.\\
 On note $\mathcal{C}$ sa représentation graphique dans un repère orthonormal.
 On prendra 2 cm pour une unité graphique.\\ 
 
\noindent$\triangleright~$\textbf{1)} Démontrer que pour tout réel $x$, on a :
 $f'(x) = g(x),~f'$ désignant la fonction dérivée de $f$.\\ 
 
 
\noindent$\triangleright~$\textbf{2)} Calculer la limite de $f$ en -~$\infty$.\\ 
 
\noindent$\triangleright~$\textbf{3) a)} Vérifier que $f(x) = x + 
\cfrac{x}{\textrm{e}^x} - \cfrac{1}{\textrm{e}^x}.$ En déduire la limite de 
$f$ en +~$\infty$. On rappelle que $\lim \limits_{x \to +~\infty} \cfrac
{\textrm{e}^x}{x} = +~\infty$.\\
\textbf{b)} Démontrer que la droite $\Delta$ d'équation $y = x$ est asymptote
à la courbe $\mathcal{C}$.\\
\textbf{c)} Étudier la position relative de la courbe $\mathcal{C}$ et de la 
droite $\Delta$.\\
On précisera les coordonnées de leur point d'intersection A.\\
 
\noindent~$\triangleright~$\textbf{4)} Donner le tableau de variation de la
 fonction $f$.\\
 
\noindent~$\triangleright~$\textbf{5)} Tracer la courbe $\mathcal{C}$ ainsi que
 la droite $\Delta$.\\ 
 
\begin{center} \textbf{Partie C} \end{center}
 
\noindent$\triangleright~$\textbf{1)} Déterminer les réels $a$ et $b$ tels
que la fonction $f$ définie sur $\R$ par :
$$F(x) = x + (ax + b)\textrm{e}^{-x}$$
 soit une primitive de $f$.\\
 
\noindent$\triangleright~$\textbf{2)} Calculer en cm$^2$ l'aire du domaine du
 plan compris entre la courbe $\mathcal{C}$, l'axe des abscisses et les droites
 d'équation $x = 1$ et $x = 3$.\\ 
En donner une valeur arrondie à $10^{-2}$ près.\\
\footnote{\scalebox{1 -1}{La Réunion juillet 1999}}
\newpage
 
%% Sujet 7 Asie juin 1999 %%
%%%%%%%%%%%%%%%%%%%%%%%%%%%%
 
\noindent  \doublebox{TES   
\hspace{2.9cm}\Large{\textbf{Baccalauréat juin 1999}} 
\hspace{2.9cm}\normalsize{}}\\
 
\noindent \textbf{Exercice 1}\hspace{2cm}4 points\\
 
\noindent Le tableau suivant recense par clinique le nombre de postes 
du personnel non médical en fonction du nombre de lits de la clinique :\\
 \small{\begin{tabular}{| c | c |c |c |c |c |c |c |c |c |c |c |}\hline
Clinique&$C_1$&	$C_2$&	$C_3$&	$C_4$&	$C_5$&
 $C_6$& $C_7$& $C_8$& $C_9$&	$C_{10}$&$ C_{11}$\\ \hline
Nombre de lits $x_i$ &122 &177&	77& 135& 109& 88& 185& 128& 120& 146& 
100\\ \hline
Nombre de postes $y_i$&	205& 249& 114&	178& 127& 122&	242& 170& 164& 
188& 172\\ \hline
\end{tabular}}\\
 
\noindent $\triangleright$\textbf{1)} Représenter le nuage de points 
associé à la série statistique $(x_i~ ;~ y_i)$ dans le plan 
rapporté à un repère orthogonal en prenant pour unités graphiques : 1 cm
pour 20 lits en abscisse et 1 cm pour 50 postes en ordonnée.\\
 
\noindent$\triangleright$\textbf{2)} Déterminer le c¦fficient de corrélation 
linéaire entre $x$ et $y$.\\
 
\noindent$\triangleright$\textbf{3)} Donner une équation de la droite de 
régression de $y$ en $x$ par la méthode des moindres carrés (les détails
des calculs ne sont pas demandés). \\ 
Pour les c¦fficients, on prendra les valeurs décimales arrondies à 
$10^{-1}$ près.\\
Tracer cette droite dans le repère précédent.\\
 
\noindent$\triangleright$\textbf{4)} Une clinique possède 35 lits.\\ 
\hspace*{0,9cm}\textbf{a)} En utilisant les résultats obtenus en 3, combien 
devrait-elle embaucher de personnel occupant un poste non médical à 
temps plein ? \\ 
\hspace*{0,9cm}\textbf{b)} En réalité, cette clinique dispose de 60 postes.\\
Calculer la différence entre le nombre de postes réels et le nombre 
de postes théoriques obtenu précédemment. \\ 
Quel pourcentage cette différence représente-t-elle par rapport 
à la situation théorique ? \\
 
\noindent  \textbf{Exercice  2}\hspace{2 cm}6 points\\
\textbf{Candidats n'ayant pas choisi l'enseignement de spécialité}\\
 
\noindent \textbf{Énoncé}\\
Un grand club de ski français propose à la vente :\\
$\bullet~$des licences ;\\ 
$\bullet~$des cartes neige à prix normal ;\\ 
$\bullet~$des cartes neige à prix réduit pour les habitants de la 
commune.\\
Pour chacun de ces titres vendus, il faut distinguer deux catégories 
: la catégorie jeunes et la catégorie adultes.\\
Le nombre de titres vendus pour la saison 98 se répartit de la manière
suivante :\\
$\bullet~$8,5 \% de licences ;\\
$\bullet~$77,5 \% de cartes neige à prix réduit ;\\
$\bullet~$1,5 \% de licences catégorie jeunes ;\\
$\bullet~$2,5 \% de cartes neige à prix normal catégorie jeunes.\\
De plus, parmi les personnes ayant acheté une carte neige à prix 
réduit, 86,5 \% sont des adultes.\\
On note :\\
L~: l'événement  « La personne a acheté une licence » ;\\
CN : l'événement « La personne a acheté une carte neige à prix 
normal » ;\\
CR : l'événement « 	  La personne a acheté une carte neige à prix
réduit » ;\\
J~ : l'événement « 	 La personne est dans la catégorie jeunes » ;\\
A~ : l'événement « 	 La personne est dans la catégorie adultes ».\\
\textbf{Questions}\\
On choisit au hasard un client de la saison 98.\\
$triangleright~$1) Déterminer la probabilité pour que :\\
\textbf{a)} cette personne ait acheté une carte neige à prix normal ;\\
\textbf{b)} cette personne ait acheté une carte neige à prix réduit 
catégorie adultes.\\
 
\noindent$\triangleright~$2) Montrer que la probabilité pour que la 
personne ait acheté une carte neige à prix réduit catégorie jeunes est
égale à 0,105.\\
 
\noindent $\triangleright~$3) Sachant que la personne a acheté une licence, 
quelle est la probabilité pour qu'elle appartienne à la catégorie adultes ?\\
 
\noindent $\triangleright~$4) Quelle est la probabilité pour que cette 
personne appartienne à la catégorie jeunes ?\\
 
\noindent $\triangleright~$5) Sachant que la personne est jeune, quelle est 
la probabilité pour qu'elle ait acheté une licence ?\\ 
Pour répondre aux questions, on peut utiliser la méthode des arbres. 
Tous les résultats sont donnés avec un arrondi à $10^{-3}$ près (ex : 0,105
ou 10,5 \%.)\\ 
 
\noindent  \textbf{Exercice 2}\hspace{2 cm}6 points\\
\textbf{Candidats ayant suivi l'enseignement de spécialité}\\
 
\noindent \textbf{Énoncé}\\
 
Dans l'espace rapporté au repère orthonormal (O 
;~$\overrightarrow{\imath} 
,~\overrightarrow{\jmath},~\overrightarrow{k}$), on a placé les 
points :\\
\begin{tabular}{l l c}
A(0 ; 0 ;  18)	& B(0 ;  15 ; 0) & \\
C(22,5 ;  0 ; 0) &	D(0 ;  0 ;  12,5)&	(voir Annexes 1 et 2)\\
E(0 ; 25 ; 0) & 	F(12,5 ;  0 ; 0). & \\
\end{tabular}\\
Le plan (ABC) a pour équation : $4 x + 6y + 5 z = 90$.\\
Le plan (DFE) a pour équation: $2x +y+ 2z = 25$.\\
La droite (GI) est l'intersection des plans (ABC) et (DFE).\\
On admet que tout point $M(x~;~ y~;~ z)$ appartenant au polyèdre 
ODGBIF a des coordonnées qui satisfont aux conditions :
$$\begin{array}{l l}
\bullet~x > 0 & \bullet~4x + 6y + 5z ¾ 90\\
\bullet~y „ 0 & \bullet~2x + y  + 2z ¾ 25\\
\bullet~z > 0 & \\
\end{array}$$
\noindent Une usine fabrique 3 types de vannes pour l'industrie 
pétrolière.\\
Pour fabriquer le modèle V1, il faut 20 heures d'usinage et 20 heures 
de montage.\\
Pour fabriquer le modèle V2, il faut 30 heures d'usinage et 10 heures 
de montage.\\
Pour fabriquer le modèle V3, il faut 25 heures d'usinage et 20 heures 
de montage.\\
Le nombre d'ouvriers spécialisés permet de disposer de 450 heures
d'usinage par semaine.\\
Le nombre d'ouvriers monteurs permet de disposer de 250 heures de
montage par semaine.\\
On désigne par $x$ le nombre de vannes de type V1 fabriquées dans une
semaine, $y$ le nombre de vannes de type V2 et $z$ le nombre de 
vannes de type V3.\\
Les points de coordonnées $(X~;~ Y~;~ Z)$ qui satisfont aux 
contraintes précédentes sont situés à l'intérieur du polyèdre ODGBIE.\\ 
 
\noindent Le bénéfice réalisé sur une vanne de type V1 est de 2 000 
F, sur une vanne de type V2, il est de 3 000 F et enfin sur une vanne de type 
V3, il est de 5 000 F.\\
Un point de coordonnées $(x~ ;~ y~ ;~ z)$ représente une production.\\
 
\noindent \textbf{Questions}\\
\textbf{a)} Montrez que les points représentant une production pour 
laquelle le bénéfice total est de 30\:000 F sont situés sur le plan (P) 
d'équation cartésienne : $2 x + 3y + 5 z = 30$.\\
Le plan (P) est tracé sur la figure de l'annexe 2.\\
\textbf{b)} Montrez qu'une production de 5 vannes de type V1, de 5 
vannes de type V2 et d'une vanne de type V3 est réalisable par cette usine en 
une semaine et que le bénéfice alors réalisé est de 30\: 000 F.\\ 
Quelle conclusion en tirez-vous sur la position du point K de 
coordonnées (5 ; 5 ; 1) ?\\
\textbf{c)} Montrez que les points représentant une production pour 
laquelle le bénéfice total est de 60\:000 F sont situés sur le plan (Q) 
d'équation cartésienne : $2 x + 3y + 5 z = 60$.\\
\textbf{d)} Quelle remarque pouvez-vous faire sur les plans (P) et (Q) ?\\
\textbf{e)} On admet que le bénéfice réalisé par l'entreprise est 
maximal lorsque le plan (R) d'équation $2 x + 3y + 5 z = b$ passe par le 
point G dont les coordonnées sont (0 ; $\cfrac{55}{7}~ ;~ \cfrac{60}{7}$).\\
Calculer ce bénéfice maximal.\\
 
\noindent \textbf{PROBLÈME}\hspace{2cm}\\
 
\noindent \large{\textbf{Partie A}}\normalsize{}\\
 
Soit $f$ la fonction définie sur [0 ; 50] par : 
$$f(x) = x^2 +	\cfrac{50x}{x + 1} - 50 \ln (x + 1) - 50.$$
La dérivée $f'(x)$ est égale à : $\cfrac{2x(x - 4)(x + 6)}{(x + 
1)^2}$.\\
La courbe ($\mathcal{C}$) de $f$ est donnée en annexe.\\
 
\noindent $\triangleright~$\textbf{1)} Étudier le signe de $f(x)$ sur 
l'intervalle [0 ; 50].\\
 
\noindent $\triangleright~$\textbf{2)} Dresser le tableau de variation de $f$ 
sur [0 ; 50] . On admet que $f(x)$ s'annule pour une seule valeur $\alpha$et de
l'intervalle ]0 ; 50[ ; en déduire le signe de $f(x)$ sur l'intervalle
[0 ; 50].\\
 
\noindent $\triangleright~$\textbf{3)} Donner un encadrement de $\alpha$ par 
deux entiers consécutifs.\\
Pour la suite du problème, on prendra pour $\alpha$ la plus petite de 
ces deux valeurs.\\
 
\noindent \large{\textbf{Partie B}}\normalsize{}\\
 
Une entreprise fabrique une quantité $x$, exprimée en kilogrammes, 
d'un certain produit.\\
Le coût marginal C, exprimé en euros, est défini sur [0 ; 50] par
$$C(x) = 2x + \cfrac{50}{x + 1}$$
 
\noindent $\triangleright$~\textbf{1)} La fonction coût total, notée 
C$_{\textrm{T}}$ est la primitive de la fonction C sur [0 ; 50] qui prend
la valeur 50 pour $x = 0$.\\
Vérifier que C$_{\textrm{T}}(x) = x^2 + 50 \ln (x + 1) + 50.$\\
 
\noindent $\triangleright~$\textbf{2)} Le coût moyen est la fonction C$_{m}$, 
définie par:
$$\textrm{C}_{m}(x) = \cfrac{\textrm{C}_{\textrm{T}} (x)}{x} 
~\textrm{sur}~ ]0 ; 50].$$
\textbf{a)} Donner une expression de C$_{m}(x)$ en fonction de $x$.\\
\textbf{b)} Vérifier que la dérivée de C$_{m}$ peut se mettre sous la forme
$$\textrm{C}'_{m}(x) = \cfrac{f(x)}{x^2}.$$
 
\noindent \large{\textbf{Partie C}}\normalsize{}\\
 
\noindent $\triangleright~$\textbf{1)} Déduire des résultats 
précédents le tableau de variation de la fonction C$_{m}$ sur ]0 ; 50].\\
 
\noindent $\triangleright~$\textbf{2)} Tracer dans un repère orthonormal (0 ;~$ 
\overrightarrow{\imath} ,~\overrightarrow{\jmath}$  ) la courbe 
représentative de C$_{m}$ sur [1 ; 50].\\
 
\noindent $\triangleright~$\textbf{3)} Quelle est la production donnant 
le coût moyen minimal ?\\
Calculer alors le coût total et le coût marginal correspondant au 
coût moyen minimal.\\
\newpage
\begin{center}
    Annexe 3\\
Courbe ($\mathcal{C}$) de la fonction $f$.\\
\psset{xunit=4mm,yunit=0.64mm}
\begin{pspicture}(20,100)
    \psline{->}(-2,80)(20,80) \psline{->}(0,0)(0,100) 
    \rput(-1,90){$y$}
    \rput(18,78){$x$}
    \rput(-1,75){O} \rput(-1,30){50} \rput(10,40){$(\mathcal{C}$)}
\pscurve(0,30)(1,21.34)(2,12.4)(3,7.19)(4,5.53)(5,7.08)(6,11.56)
(7,18.78)(8,28.58)(9,40.87)(10,55.56)(11,72.588)(12,91.906)
\end{pspicture}
\end{center}
 \footnote{\scalebox{1 -1}{Asie juin 1999}}
\newpage
%%%%%% Sujet 8 National remplacement %%%
%%%%%%% septembre 1998      %%%%%%%%%%%%
\noindent  \doublebox{TES   
\hspace{2.9cm}\Large{\textbf{Baccalauréat juin 1999}} 
\hspace{2.9cm}\normalsize{}}\\
 
\noindent \textbf{Exercice 1}\hspace{2cm}5 points\\
 
On s'intéresse à l'évolution de la population mondiale entre les 
années 1950 et 1990. Le document ci-après donne une représentation graphique
des données pour les années 1950, 1960, 1970, 1980 et 1990 en papier 
semi-logarithmique.\\
L'allure du graphique incite à chercher un modèle sous la forme 
d'une fonction $f$ définie par : 
$$f(t) = A \textrm{e}^{at}$$
où $t$ désigne le rang de l'année, avec comme origine des temps 
l'année 1950, et $f(t)$ la population en milliards d'habitants.\\
 
\noindent $\triangleright~$\textbf{1)} Déterminer les c¦fficients $A$ 
et $a$ en utilisant les données de 1950 et de 1990, à savoir : 
\begin{center} \begin{tabular}{| l *{2}{| c}|}\hline
\textbf{Rang} $t$ & 0 & 	40\\ \hline
\textbf{Population en milliards d'habitants} & 2,5 & 	5,2\\ \hline
\end{tabular}\\
\end{center}
On donnera les valeurs exactes de $A$ et $a$ puis des valeurs 
approchées à $10^{-~4}$ près.\\
Dans la suite on considérera que : $f(t) = 2,5 \textrm{e}^{0,018t}$.\\
 
\noindent $\triangleright~$\textbf{2)} Représenter graphiquement $f$ 
dans le même repère semi-logarithmique que le nuage (document page suivante). 
Justifier le tracé.\\ 
 
\noindent $\triangleright~$\textbf{3)} À l'aide du modèle proposé, 
calculer une estimation de l'année au cours de laquelle la population mondiale 
devrait dépasser 10 milliards d'habitants. Indiquer sur le graphique comment 
contrôler ce résultat.\\
 
\noindent $\triangleright~$\textbf{4)} Calculer $\cfrac{f(t + 1) - 
f(t)}{f(t)}$.\\
Donner la valeur exacte, puis une valeur approchée.\\
Interpréter ce résultat en terme de taux de croissance annuel.\\
 
\begin{center} \begin{pspicture}(10,6)
\rput(-1,4.605){100} \rput(-1,2.303){10} \rput(-1,0){1}
\rput(0,-0.6){1950} \rput(1,-0.6){1960} \rput(2,-0.6){1970} 
\rput(3,-0.6){1980} \rput(4,-0.6){1990} \rput(5,-0.6){2000} 
\rput(6,-0.6){2010} \rput(7,-0.6){2020} \rput(8,-0.6){2030} 
\rput(8,4.7){Population mondiale} \rput(4,-0.5){Année}
\rput(0,0.916){*} \rput(1,1.13){*} \rput(2,1.25){*} \rput(3,1.504){*} 
 \rput(4,1.649){*} 
\psline(0,0)(10,0)  \psline(0,0)(0,4.5) \psline(0,0.693)(10,0.693) 
\psline(0,1.099)(10,1.099)
 \psline(0,1.386)(10,1.386) \psline(0,1.609)(10,1.609) 
\psline(0,1.792)(10,1.792)
 \psline(0,1.946)(10,1.946) \psline(0,2.079)(10,2.079) 
\psline(0,2.197)(10,2.197) \psline(0,2.303)(10,2.303) 
\psline(0,2.996)(10,2.996)
 \psline(0,3.401)(10,3.401) \psline(0,3.689)(10,3.689) 
\psline(0,3.912)(10,3.912)
 \psline(0,4.094)(10,4.094) \psline(0,4.249)(10,4.249) 
\psline(0,4.382)(10,4.382)
\psline(0,4.5)(10,4.5) \psline(0,4.605)(10,4.605)
\psline(1,0)(1,0.1) \psline(2,0)(2,0.1) \psline(3,0)(3,0.1) 
\psline(4,0)(4,0.1) 
\end{pspicture}
\end{center}
\vspace*{1cm}
\noindent \textbf{Exercice 2}\hspace{2cm}5 points\\
\textbf{(obligatoire)}
 
Dans cet exercice on pourra utiliser les notations usuelles $p(E)$ pour désigner la
probabilité d'un événement E,\ $p$(F/E) ou $p_{\textrm{E}}$(F) pour
désigner la probabilité conditionnelle de F, sachant l'événement E 
réalisé.\\
Un concours de recrutement de techniciens hautement qualifiés est ouvert 
uniquement aux étudiants de deux écoles ; l'une s'appelle l'école Archimède,
l'autre l'école Ptolémée.\\
On dispose des informations suivantes concernant les taux de réussite à ce
concours pour l'année 1997 :\\
- le taux de réussite pour les candidats issus de l'école Archimède est 
de :~85\: \%~;\\
- le taux de réussite pour les candidats issus de l'autre école est 
de :~ 80\: \%~;\\
- le taux de réussite pour l'ensemble des candidats est de :~ 82\: \%.\\
On peut interpréter ces données en termes probabilistes ; on suppose pour
cela qu'on choisit un candidat au hasard.\\
On note R l'événement : « le candidat a réussi ».\\
On note de même A l'événement : « le candidat est issu de l'école 
Archimède ».\\
On note $\overline{\textrm{R}}$ et $\overline{\textrm{A}}$ les événements
contraires de R et de A.\\
 
\noindent $\triangleright~$\textbf{1)} Interpréter les données numériques de
l'énoncé en termes  probabilistes.\\
 
\noindent $\triangleright~$\textbf{2)} Les événements R et A sont-ils
indépendants ? Justifier votre réponse.\\
 
\noindent $\triangleright~$\textbf{3)} L'objet de cette question est de 
déterminer la proportion de candidats issus de l'école Archimède parmi les
candidats.\\ 
On note $x$ la proportion de candidats issus de l'école Archimède parmi les
 candidats : c'est aussi la probabilité qu'un candidat, choisi au hasard, soit
 un candidat issu de l'école Archimède.\\ 
\hspace*{0,9cm}\textbf{a)} Exprimer $p$(R $\cap$ A),~ p($\overline{\textrm{A}})$
et $p$(R $\overline {\textrm{A}})$ en fonction de $x$.\\
\hspace*{0,9cm}\textbf{b)} En déduire l'expression de $p$(R) en fonction de $x$.\\
\hspace*{0,9cm}\textbf{c)} Déterminer la valeur de $x$.\\  
 
\noindent \textbf{Exercice 2}\hspace{2cm}5 points\\
\textbf{(spécialité)}
 
\textit{Les deux questions} 1) et 2) \textit{peuvent être traitées 
indépendamment l'une de l'autre}.\\
 
\noindent $\triangleright~$\textbf{1)} On envisage un jeu 
publicitaire sous la forme d'un QCM (questionnaire à choix multiples).\\
Il comporte quatre questions et, pour chaque question, trois réponses 
sont possibles dont une seule exacte.\\
Un joueur répond en choisissant au hasard une réponse pour chaque 
question.\\
\textbf{a)} De combien de façons différentes peut-il remplir le 
questionnaire ?\\
\textbf{b)} On nomme $X$ la variable aléatoire égale au nombre de 
réponses exactes obtenues par le joueur. Donner la loi de probabilité de 
$X$.\\
 
\noindent $\triangleright~$\textbf{2)} Pour accroître la difficulté, 
on modifie le QCM : il comporte cette fois cinq questions et, pour chaque 
question, quatre réponses sont possibles dont une seule exacte.\\
Un joueur remplit au hasard le QCM.\\
La deuxième ligne du tableau ci-dessous indique les probabilités 
respectives pour que le joueur ait exactement 0, 1, 2, 3, 4, 5 réponses justes.\\
\begin{center}%\setlength{\extrarowheight}{5pt}
\begin{tabular}{| l *6{| c}|}\hline
Nombre de bonnes réponses &	0&	1&	2&	3&	4&	5\\ \hline
Probabilité correspondante &	$\cfrac{243}{1024}$& $\cfrac{405}{1024}$ 
&$\cfrac{270}{1024}$&$\cfrac{90}{1024}$&$\cfrac{15}{1024}$ & 
$\cfrac{1}{1024}$\\ 
\hline
Nombre de points obtenus &  &  &  &  & 	$16 - x$& 	20\\ \hline
\end{tabular}\\
\end{center}
Il est prévu d'attribuer 4 points par réponse juste, on ne sait 
comment pénaliser une réponse fausse : on note $x$ le nombre entier de points
retirés au joueur par réponse fausse.\\ 
\noindent \textbf{a)} Recopier le tableau ci-dessus et compléter la dernière 
ligne, en indiquant dans chaque cas le nombre de points obtenus en fonction de
$x$. On définit ainsi une variable aléatoire $N$ égale au nombre de points
obtenus par le joueur.\\
\textbf{b)} Exprimer l'espérance de $N$ en fonction de $x$.\\
 
\noindent \textbf{Problème}\hspace{2cm}10 points\\
 
Une entreprise spécialisée produit deux types de détergents liquides qu'on
nommera A et B pour simplifier.\\
Les deux parties du problème sont indépendantes.\\
 
\noindent \textbf{Partie A}\\
 
La  courbe ci-dessous  représente le coût total de production du produit $A$
en fonction de la quantité produite. On note $x$ la quantité produite 
exprimée en litres et C$_{\textrm{T}}(x)$ le coût total exprimé en francs,
$x$ variant de 0 à  800.\\
 On notera que C$_{\textrm{T}}(0) = 0,~ \textrm{C}_{\textrm{T}}(450) = 400,~ 
 \textrm{C}_{\textrm{T}} (800) =\:800$ et que la tangente au point d'abscisse 450
 passe par l'origine O du repère.\\
 \begin{center} \begin{pspicture}(0,0)(8,4.5)
     \rput{90}(0,-0.7){0}  \rput{90}(1,-0.7){100}  
     \rput{90}(2,-0.7){200} 
      \rput{90}(3,-0.7){300}  \rput{90}(4,-0.7){400}  
      \rput{90}(5,-0.7){500} 
       \rput{90}(6,-0.7){600}  \rput{90}(7,-0.7){700}  
       \rput{90}(8,-0.7){800}
       \psline{->}(0,0)(8,0)  \psline{->}(0,0)(0,4.5)
       \rput(-0.8,0){0}  \rput(-0.8,1){400}  \rput(-0.8,2){800} 
        \rput(-0.8,3){1200}  \rput(-0.8,4){1600}  \rput(-0.8,4.5){1800}
	\psline{<->}(3,0.667)(6,1.333) \psline[linestyle=dashed](0,0)(3,0.667)
	\psline[linestyle=dashed](0,4.5)(8,4.5) 
	\psline[linestyle=dashed](0,1)(4.5,1)
	\psline[linestyle=dashed](4.5,0)(4.5,1)
	\psline[linestyle=dashed](8,0)(8,4.5) 
	\pscurve(0,0)(1,0.6)(2,0.8)(3,0.9)(4,0.925)(4.5,1)(5,1.15)(6,1.7)
	(7,2.6)(8,4.5)
	\end{pspicture}
	\end{center}
	\vspace{1.5cm}
 \textit{Répondre aux questions suivantes en utilisant les informations portées
 sur ce graphique.}\\
 
\noindent $\triangleright~$\textbf{1)} Les économistes définissent 
le coût marginal comme le supplément de coût de production engendré par
la production d'une unité supplémentaire. On considère qu'il peut 
être modélisé par la dérivée du coût total. Nous le
 noterons C$_m$. On a donc C$_m = \textrm{C}'_{\textrm{T}}$. Parmi les quatre
 graphiques  (1,~2,~3 et 4) de la feuille jointe, un correspond au 
 coût marginal associé à la production du détergent A.
 Lequel ? Justifier la réponse.\\
 \begin{center}
     \begin{tabular}{ c c }
     \psset{xunit=0.05mm,yunit=0.5mm}
     \begin{pspicture}(-10,-15)(900,15)
	 \rput{90}(0,-10){0} \rput{90}(100,-10){100} \rput{90}(200,-10){200} 
	 \rput{90}(300,-10){300} \rput{90}(400,-10){400} \rput{90}(500,-10){500} 
	 \rput{90}(600,-10){600} \rput{90}(700,-10){700} \rput{90}(800,-10){800}
	 \rput(-70,-15){-15} \rput(-70,-10){-10} \rput(-70,-5){-5} 
	 \rput(-70,0){0} \rput(-70,5){5} \rput(-70,10){10}
	 \psline(0,-12.5)(800,10) \psline(0,0)(800,0) \psline(0,-15)(0,10)  
	 \rput(300,15){graphique 1}
	 \end{pspicture}
	 &\psset{xunit=0.05mm,yunit=1mm}
 \begin{pspicture}(0,0)(800,15)
\psline(0,0)(800,0)  \psline(0,0)(0,15)
\rput{90}(0,-5){0} \rput{90}(100,-5){100} \rput{90}(200,-5){200} 
\rput{90}(300,-5){300} \rput{90}(400,-5){400} \rput{90}(500,-5){500} 
\rput{90}(600,-5){600} \rput{90}(700,-5){700} \rput{90}(800,-5){800}
\rput(-70,0){0} \rput(-70,5){5} \rput(-70,10){10}
\pscurve(0,2.5)(100,0.71)(300,0)(400,1.08)(500,3.125)(600,6.125)
(700,10.083)(800,15)
\rput(300,15){graphique 3}	 
	 \end{pspicture}\\
	 ~&~\\
	 ~& ~\\
\psset{xunit=0.05mm,yunit=4mm}
\begin{pspicture}(0,0)(900,5)
\rput{90}(0,-1){0} \rput{90}(100,-1){100} \rput{90}(200,-1){200} 
\rput{90}(300,-1){300} \rput{90}(400,-1){400} \rput{90}(500,-1){500}
\rput{90}(600,-1){600} \rput{90}(700,-1){700} \rput{90}(800,-1){800}
\rput(-100,0){0} \rput(-100,5){5}
\psline(0,1)(800,1)
\psline(0,0)(800,0) \psline(0,0)(0,5)
\rput(300,5){graphique 2}
\end{pspicture}
&\psset{xunit=0.05mm,yunit=2mm}
\begin{pspicture}(-100,0)(900,10)
    \rput{90}(0,-2){0} \rput{90}(100,-2){100} \rput{90}(200,-2){200} 
\rput{90}(300,-2){300} \rput{90}(400,-2){400} \rput{90}(500,-2){500}
\rput{90}(600,-2){600} \rput{90}(700,-2){700} \rput{90}(800,-2){800}
\rput(-100,0){0} \rput(-100,5){5} \rput(-100,10){10} 
\psline(0,0)(800,0) \psline(0,0)(0,10)
\pscurve(0,7.5)(100,9)(250,9.75)(350,9.75)(600,7)(800,1.5)
\rput(300,11){graphique 4}
 \end{pspicture}
	 \end{tabular}\\
	 \end{center}
	 \vspace{1.5cm}
 
\noindent $\triangleright~$\textbf{2)} Déterminer $\int_{0}^{450}\: 
\textrm{C}_{m}(x)\:dx$.\\
 
 \noindent \textbf{Partie B}\\
 
Pour le détergent B l'entreprise est en situation de monopole. Une étude a
 permis de modéliser le coût moyen de production par :\\ 
$$f(x) = 0,5x + \cfrac{8}{x}~ \textrm{où}~ x > 0.$$\\
Le coût moyen $f(x)$ est exprimé en milliers de francs et la quantité
produite $x$ en hectolitres. On note $\mathcal{C}$ la courbe représentative de
la  fonction $f$ dans un repère orthonormé du plan (unité graphique : 1 cm).\\
 
\noindent $\triangleright~$\textbf{1) Étude de la fonction coût moyen}\\
\hspace*{0,9cm}\textbf{a)} Étudier le sens de variation de cette fonction sur
l'intervalle $]0,~ +~\infty[$.\\
\hspace*{0,9cm}\textbf{b)} Déterminer les limites de $f(x)$ en 0 et $+~\infty$.\\
\hspace*{0,9cm}\textbf{c)} Montrer que la droite D d'équation $y = 0,5x$ est
asymptote à la courbe $\mathcal{C}$. Étudier la position relative de
$\mathcal{C}$ par rapport à D.\\ 
\hspace*{0,9cm}\textbf{d)} Construire $\mathcal{C}$ ainsi que D, donner un
tableau de valeurs.\\
 
\noindent $\triangleright~$\textbf{2) Seuils de rentabilité pour l'entreprise}\\
L'entreprise ne peut être bénéficiaire que si le prix de vente de
l'hectolitre est supérieur au coût moyen de fabrication.\\ 
Le prix de vente de l'hectolitre $p(x)$ est fonction de la quantité $x$ 
vendue.
$$p(x) = - 0,8x + 13$$
 où $p(x)$ est exprimé en milliers de francs et $x$ en hectolitres.\\ 
\hspace*{0,9cm}\textbf{a)} On note $\mathcal{P}$ la représentation graphique de
la fonction $p$. Tracer $\mathcal{P}$ dans les mêmes axes que la représentation
de $f$, puis déterminer graphiquement l'intervalle dans lequel doit se situer
la production $x$ pour que l'entreprise soit bénéficiaire.\\ 	
\hspace*{0,9cm}\textbf{b)} Retrouver le résultat précédent par le calcul.
(On pourra se ramener à une inéquation du second degré).\\
 \footnote{\scalebox{1 -1}{France métropolitaine remplacement  de septembre 1998}}
\newpage
 
%%%% Sujet 9 Amérique du Sud %%%%
%%%%         novembre 1998   %%%%
 
\noindent  \doublebox{TES   
\hspace{1.9cm}\Large{\textbf{Baccalauréat novembre 1998}} 
\hspace{1.9cm}\normalsize{}}\\
 
\noindent \textbf{Exercice 1}\hspace{2cm}5 points\\
 
\noindent \textbf{I)}  Le tableau ci-dessous indique les pourcentages d'accès
au niveau baccalauréat d'une génération d'élèves. 
\begin{center}\small{} \begin{tabular}{|p{2.5cm} *8{| c}|}\hline
\textbf{Année} $x_{i}$ &1980 & 1982& 1984& 1986& 1988& 1990& 1992& 
1994\\ \hline 
\small{\textbf{Taux d'accès au niveau baccalauréat} $y_{i}$} &  34 \%& 
37,5\% &  35,8\%& 39,8 \%& 46,3 \%& 56,1 \%& 62,5 \%& 70,7 \%\\ \hline
\end{tabular}\\
\end{center}
\begin{flushright} \small{Source : d'après un document du Ministère de
    l'Éducation Nationale} \end{flushright}
N.B.  \textit{Les calculs statistiques seront effectués à la machine, aucun 
détail n'est demandé dans cette partie.}\\ 
 
\noindent $\triangleright~$\textbf{1) a)} Représenter le nuage de points
associé à la série statistique $(x_{i},~ y_{i})$ dans le plan rapporté à un
repère orthogonal :\\
- sur l'axe des abscisses on placera 1980 à l'origine et on choisira 
1 cm pour une année ;\\
- sur l'axe des ordonnées on placera 30 à l'origine et on choisira 1 cm pour
2 \%.\\
\textbf{b)} Calculer les coordonnées du point moyen G associé à cette série 
double et placer ce point sur le graphique précédent.\\
 
\noindent $\triangleright~$\textbf{2) a)} Calculer le c¦fficient de 
corrélation linéaire entre $x$ et $y$. Peut-on envisager un ajustement 
affine ? \\
\textbf{b)} Déterminer une équation de la droite de régression D de 
$y$ en $x$ : on prendra la valeur approchée à trois décimales par défaut
pour le c¦fficient directeur de la droite et l'arrondi à l'unité pour l'autre 
c¦fficient.\\ 
\textbf{c)} Tracer la droite D sur le graphique de la question 
\textbf{1) a)} en expliquant sa construction.\\
 
\noindent $\triangleright~$\textbf{3)} En supposant que l'évolution 
ait été la même pour les années suivantes, donner une estimation du taux 
d'accès au niveau baccalauréat pour 1996.\\
 
\noindent \textbf{II)}  Lors de la publication du tableau de la partie 
\textbf{I)}, le taux d'accès au niveau baccalauréat pour 1996 n'était pas
encore connu. On l'a connu seulement plus tard.\\ 
 
\noindent $\triangleright~$\textbf{1)} Déterminer le taux d'accès en 
1996 si l'on sait que, pour la période 1980 (incluse) à 1996 (incluse), la 
moyenne de ce taux est exactement de 50 \%, en ne retenant que les années 
paires.\\ 
 
\noindent $\triangleright~$\textbf{2)} Comparer alors avec l'estimation faite à
la question \textbf{3)} de la partie \textbf{I} et donner en pourcentage l'erreur
commise en remplaçant la valeur exacte par l'estimation faite. \\
 
 
\noindent \textbf{Exercice 2}\hspace{2cm}5 points\\
\textbf{(obligatoire)}\\
 
Dans une grande ville, une maladie à incubation lente touche 0,1 \% de 
la population. Un test de dépistage est proposé :\\
- lorsqu'une personne est malade, le test est positif dans 95 \% des 
cas et négatif dans 5 \% des cas ;\\
- lorsqu'une personne n'est pas malade, le test est négatif dans 96 \% 
des cas, mais déclare la personne malade, c'est-à-dire est positif, dans 
4 \% des cas.\\
Lorsqu'une personne, prise au hasard, passe le test, on note
- M l'événement « la personne est malade » ;\\
- $\overline{\textrm{M}}$ l'événement « la personne n'est pas malade » ;\\
- T l'événement « le test est positif » ;\\
- $\overline{\textrm{T}}$ l'événement « le test est négatif ».\\
 
\noindent $\triangleright~$\textbf{1)}	Donner la valeur de la 
probabilité $p$(M) et les valeurs des probabilités conditionnelles
suivantes : $p$(T/M), $p$(T/$\overline{\textrm{M}}$), $p$($\overline{\textrm{T}}$
/M) et $p$($\overline{\textrm{T}}$/$\overline{\textrm{M}}$).\\
 
\noindent $\triangleright~$\textbf{2) a)} Calculer la probabilité de 
l'événement « M et T », notée $p$(M $\cap$ T).\\ 
\textbf{b)} Calculer la probabilité de l'événement « M et T », notée 
$p$(M~ $\cap$~ T).\\ 
\textbf{c)} En déduire que la probabilité de T vaut $p$(T) = 0,040\: 91.\\ 
 
\noindent $\triangleright~$\textbf{3)} Calculer la probabilité pour 
que le test donne un résultat non conforme à la réalité.\\ 
 
\noindent $\triangleright~$\textbf{4)} Le maire de la ville passe le 
test : il est positif. Donner la probabilité, à $10^{-1}$ près, que le maire
soit effectivement malade.\\
 
 
\noindent \textbf{Exercice 2}\hspace{2cm}5 points\\
\textbf{(spécialité)}\\
 
À partir de 1997 une association d'aide à la recherche médicale 
envoie chaque année à Monsieur X un courrier pour l'inviter à l'aider 
financièrement par un don. Monsieur X a répondu favorablement en 1997 
en envoyant un don. On admet que, chaque année è partir de 1998, la 
probabilité pour que Monsieur X fasse un don est égale à 0,9 s'il a 
fait un don l'année précédente et à 0,4 s'il n'a rien donné l'année 
précédente.\\
On note pour tout entier naturel $n$ :\\
- E$_{n}$ l'événement : « Monsieur X est donateur en 1998 + $n$ » ;\\
- P$_{n}$ la probabilité de E$_{n}$ ;\\
- $\overline{\textrm{E}_{n}}$ l'événement contraire de E$_{n}$.\\ 
 
\noindent $\triangleright~$\textbf{1)} Traduire les données en termes 
de probabilités conditionnelles concernant les événements E$_{n+1}$,~E$_{n}$,
$\overline{\textrm{E}_{n}}$.\\ 
 
\noindent $\triangleright~$\textbf{2) a)} Préciser la valeur de 
P$_{0}$.\\		
\textbf{b)} Calculer P(E$_{1}~  \cap~ $ E$_{0}$) et P(E$_{1}~ \cap~ 
\overline{\textrm{E}_{0}}$). En déduire la valeur de P$_{1}$.\\	
 
\noindent $\triangleright~$\textbf{3) a)} Montrer que P(E$_{n+1} \cap  
\textrm{E}_{n}$) = 0,9P$_{n}$ et que P(E$_{n+1}~ \cap~ 
\overline{\textrm{E}_{n}})$ = 0,4(1 - P$_{n}$) pour tout entier $n$.\\
\textbf{b)} En déduire que P$_{n+1} = 0,5\textrm{P}_{n} + 0,4$ pour tout
entier naturel $n$.\\
\textbf{c)} Quelle est la probabilité pour que Monsieur X soit donateur en 
2001 ?\\
 
\noindent $\triangleright~$\textbf{4)} On définit une suite (U$_{n}$) en 
posant pour tout entier naturel $n : \textrm{U}_{n} = \textrm{P}_{n} - 
0,8$\\
\textbf{a)} Démontrer que la suite (U$_{n}$) est géométrique. Préciser sa
raison et son premier terme.\\
\textbf{b)} Exprimer (U$_{n}$) en fonction de $n$.\\
\textbf{c)} En déduire que P$_{n} = 0,1 \times 0,5^n + 0,8$ pour tout entier
naturel $n$.\\		
\textbf{d)} Déterminer la limite de la suite (P$_{n})$.\\
 
\noindent \textbf{Problème}\hspace{2cm}10 points\\
 
Sur le graphique ci-après, sont tracées dans un repère orthogonal, 
les courbes représentatives $\mathcal{C}_{f}$ et $\mathcal{C}_{g}$ de deux
fonctions $f$ et $g$, dérivables sur l'intervalle [0 ; + ~$\infty$[.\\
 
\noindent \textbf{Partie A - Question préliminaire} (les résultats seront 
donnés à 0,1 près).\\
 
\noindent $\triangleright~$\textbf{1)} Résoudre graphiquement les 
équations $f(x) = 7$ et $f(x) = 4$.\\
 
\noindent $\triangleright~$\textbf{2)} Lire graphiquement $g(0)$.\\
 
\noindent $\triangleright~$\textbf{3)} Dresser le tableau de 
variation de la fonction $f$ sur l'intervalle [0 ; 14].\\
 
\noindent $\triangleright~$\textbf{4)} En déduire le signe de $f'$ sur 
l'intervalle [0; 14], où $f'$ désigne la fonction dérivée de la 
fonction $f$.\\
 
\noindent \textbf{Partie B}\\
 
La fonction $f$ est la fonction de demande d'un produit, elle met en 
correspondance le prix $f(x)$ du produit et la quantité $x$ achetée par 
les consommateurs.\\ 
La fonction $g$ est la fonction d'offre, elle met en correspondance le 
prix $g(x)$ du produit et la quantité $x$ vendue par les producteurs. La 
quantité est exprimée en milliers d'unités et le prix en centaines de 
francs.\\ 
 
\noindent $\triangleright~$\textbf{1) Interprétation économique}\\
À l'aide de la lecture graphique faite en \textbf{A}, répondre aux questions 
suivantes :\\
\textbf{a)} Quelle quantité est achetée par les consommateurs :\\
- si le prix est de 700 F ?\\
- si le prix est de 400 F ?\\ 
\textbf{b)} Au-dessous de quel prix les producteurs ne sont-ils pas 
prêts à  vendre ?\\
 
\noindent $\triangleright~$\textbf{2) Étude de la recette marginale}\\
La fonction recette R est définie sur l'intervalle [0 ; 14] par R$(x) = 
x f(x)$.\\
Une valeur approchée de la recette marginale (recette pour le 
$x^{\textrm{e}}$ produit vendu) est donnée par R'$(x)$, où R' est la fonction
dérivée de la fonction R.\\ 
On remarque que pour tout réel $x$ de l'intervalle [0 ; 14], R$'(x) = 
f(x) + x f'(x)$.\\ 
\textbf{a)} Déduire du \textbf{A - 4)} le signe de R$'(x) - f(x)$ sur
l'intervalle [0 ; 14].\\
\textbf{b)} Comparer alors, pour tout niveau de production, la recette 
marginale et le prix de vente $f(x)$.\\
 
\noindent $\triangleright~$\textbf{3) Équilibre du marché}\\
\textbf{a)} La fonction $f$ représentée sur le graphique est définie sur 
l'intervalle [0 ; +~$\infty$[~ par : 
$$f(x) = \cfrac{40}{x + 2}.$$ 
Calculer $\lim\limits_{x \to +~\infty} f(x)$. Quelle interprétation économique
peut-on faire de ce résultat ?\\
\textbf{b)} La fonction $g$ représentée sur le graphique est définie sur 
l'intervalle [0; + ~$\infty$[ par :
$g(x) = \cfrac{1}{18}x^2 + 3$.\\
Dans un marché à concurrence pure et parfaite, le prix $p_{0}$ qui
se forme sur le marché selon la « loi de l'offre et de la demande » 
correspond à l'égalité de l'offre et de la demande, c'est-à-dire à l'ordonnée
du point d'intersection I des deux courbes $\mathcal{C}_{f}$ et 
$\mathcal{C}_{g}$.\\
Soit $x_{0}$ l'abscisse du point d'intersection I.\\
- Montrer par le calcul que $x_{0}$ est solution de l'équation \\
$$(\textrm{E})~~ x^3 + 2x^2 + 54x - 612 = 0.$$
- Développer l'expression $(x - 6) (x^2 + 8x + 102)$, résoudre 
l'équation (E), et en déduire la valeur de $x_{0}$.\\
- Calculer $p_{0} = f(x_{0})$.\\
 
\noindent $\triangleright~$\textbf{4) Le surplus des consommateurs}\\ 
Le surplus des consommateurs se définit comme la différence entre\\
-  le montant maximal que les consommateurs auraient été prêts à payer 
pour acheter une quantité $x_{0}$ et le montant qu'ils payent 
effectivement.\\
Ce nombre S$_{\textrm{C}}$, en situation de concurrence pure et 
parfaite, est donné en centaine de milliers de francs par : 
$$\textrm{S}_{\textrm{C}} = \int\limits_{0}^{x_{0}}\:f(x)\:\textrm{d}x - p_{0}x_{0}.$$
On prendra $x_{0} = 6$ et $p_{0} = 5$.\\ 
\textbf{a)} Calculer S$_{\textrm{C}}$.\\
\textbf{b)} Soit les points O(0 ; 0), P($x_{0}$~; 0),~ I($x_{0}~; 
~p_{0}$) et R(0~;~ $p_{0}$).\\
Sachant que le produit $p_{0} \times x_{0}$ est représenté par l'aire du
rectangle OPIR, interpréter graphiquement le surplus des consommateurs. 
\begin{center}
    \psset{xunit=5mm,yunit=5mm}
    \begin{pspicture}(0,0)(18,21)
	\psgrid[gridlabelcolor=white]
	\psline{->}(0,0)(1,0) \psline{->}(0,0)(0,1)
	\rput(0,-0.8){0} \rput(5,-0.8){5} \rput(10,-0.8){10} \rput(14,-0.8){14}
	\rput(-0.8,5){5} \rput(-0.8,10){10} \rput(-0.8,15){15} 
	\rput(-0.8,20){20}  
	\rput(0.5,-0.4){$\overrightarrow{\imath}$}
        \rput(-0.4,0.5){$\overrightarrow{\jmath}$}
	\rput(15,1.8){$\mathcal{C}_{f}$}  \rput(14,13){$\mathcal{C}_{g}$}
\psplot[plotpoints=1000,linecolor=gray]{0}{15}{40 x 2 add div}
\psplot[plotpoints=1000]{0}{14}{x dup mul 18 div 3 add}
\end{pspicture}
\end{center}
 \footnote{\scalebox{1 -1}{Amérique du Sud novembre 1998}}\\
\newpage
 
%%%%%%%%  Sujet 10 Nouvelle-Calédonie %%%%%%
%%%%%%%%%% décembre 1998 %%%%%%%%%%%%
 
\noindent  \doublebox{TES   
\hspace{1.9cm}\Large{\textbf{Baccalauréat décembre 1998}} 
\hspace{1.9cm}\normalsize{}}\\
 
\noindent \textbf{Exercice 1}\hspace{2cm}4 points\\
 
Au cours d'une kermesse, l'animateur d'un stand dispose, dans un 
enclos, de douze cages peintes : sept sont blanches, deux noires et 
les trois autres vertes. L'animateur place alors une souris dans 
l'enclos. On suppose qu'à chaque jeu, la souris choisit d'entrer au 
hasard dans une cage et que tous les choix sont équiprobables.\\ 
Un joueur participe au jeu. Le règlement du jeu est le suivant :\\
- si la souris entre dans une cage blanche, le joueur perd ; \\
- si la souris entre dans une cage noire, le joueur gagne ;\\
- si la souris entre dans une cage verte, l'animateur remet la souris 
dans l'enclos ; si la souris entre alors dans une cage noire, le 
joueur gagne, sinon il perd.\\
On suppose que le choix de la deuxième cage est indépendant du choix de la
première.\\ 
 
\noindent $\triangleright~$\textbf{1)} Montrer que la probabilité de 
l'événement « le joueur gagne » est	$\cfrac{5}{24}$
 
\noindent $\triangleright~$\textbf{2)} Un joueur possède 10 F qu'il 
verse pour participer à une partie. S'il gagne, il reçoit $k$ francs ; 
sinon, il ne reçcoit rien. Soit $X$ la variable aléatoire prenant pour 
valeur la somme que possède le joueur après la partie.\\ 
\textbf{a)} Déterminer la loi de probabilité de la variable 
aléatoire $X$.\\
\textbf{b)} Calculer, en fonction de $k$, l'espérance mathématique E($X$) de
la variable aléatoire $X$.\\
\textbf{c)} Quelle valeur faut-il donner à $k$ pour que le jeu soit équitable 
(c'est-à-dire pour que ce joueur puisse espérer posséder 10 F à la 
fin de la partie) ?\\
 
\noindent \textbf{Exercice 2}\hspace{2cm}5 points\\
\textbf{(obligatoire)}\\
 
Une lessive est vendue habituellement, dans les magasins A et B, par 
barils de 5 kg, au prix de 65 F le baril.\\ 
 
\noindent $\triangleright~$\textbf{1)} On suppose que cette lessive 
est en promotion dans ces deux magasins :\\
\textbf{a)} Dans le magasin A, on fait une réduction de 10 \% sur le prix du
baril.\\ 
Dans le magasin B, on offre 10 \% de produit gratuit en plus pour 
l'achat d'un baril.\\ 
Déterminer dans lequel des deux magasins il est le plus avantageux 
d'acheter cette lessive.\\
\textbf{b)} Répondre à la même question si, dans A, on fait une réduction de 
20 \% et, dans B, on offre 25 \% de produit gratuit en plus.\\
 
\noindent $\triangleright~$\textbf{2)} On suppose maintenant que, 
dans le magasin A, on fait une réduction de $x$ \% du prix du baril et 
que, dans le magasin B, on offre $y$ \% de produit gratuit en plus pour 
l'achat d'un baril.\\ 
\textbf{a)} Quelle relation doivent vérifier $x$ et $y$ pour que les promotions 
soient les mêmes dans les deux magasins ?\\ 
Dans ces conditions, déterminer $x$ lorsque $y$ = 25.\\
\textbf{b)} Dans cette question, $x = 10$. Quel pourcentage minimum, en nombre 
entier, de produit gratuit doit offrir le magasin B pour que sa 
promotion soit plus avantageuse que celle de A ?\\ 
 
\noindent \textbf{Exercice 2}\hspace{2cm}5 points\\
\textbf{(spécialité)}\\
 
Une observation faite sur la fréquentation d'un stade de football a 
permis de constater, pour chaque année, un taux de réabonnement de 80 \%, ainsi 
que l'apparition de 4000 nouveaux abonnés.\\
\textit{L'objet de cet exercice est l'étude du devenir du nombre annuel des 
abonnés, en supposant que la situation décrite par l'observation reste la 
même au fil des ans.\\
Les questions }\textbf{2)} \textit{et} \textbf{3)} \textit{peuvent être 
traitées indépendamment l'une de l'autre.}\\
On note $a_{n}$ le nombre des abonnés à la fin de la $n^{\textrm{e}}$ année
et on précise que $a_{0}$ = 7 000.\\ 
 
\noindent $\triangleright~$\textbf{1)} Expliquer pourquoi, pour tout 
nombre entier naturel $n$, on a $a_{n+1} = 0,8 a_{n} + 4000$.\\ 
 
\noindent $\triangleright~$\textbf{2)} L'objet de cette question est 
l'étude graphique de la suite ($a_{n}$).\\
On considère un repère orthonormal (unité graphique : 0,5 cm représente
1000 abonnés).\\
\textbf{a)} Tracer dans ce repère la droite (D) d'équation $y = 0,8x + 
4000$ et la droite ($\Delta$) d'équation $y = x$, pour les abscisses comprises
entre 0 et 25\:000.\\
\textbf{b)} Placer $a_{0}$ sur l'axe des abscisses. Utiliser les droites 
précédentes pour placer sur l'axe des abscisses les valeurs $a_{1},~ 
a_{2}$ et $a_{3}$.\\
\textbf{c)} Si l'on poursuit le processus graphique précédent, quelle limite 
peut-on présumer pour la suite ($a_{n}$) ?\\
 
\noindent $\triangleright~$\textbf{3)} L'objet de cette question est l'étude
numérique de la suite ($a_{n}$).\\ 
Soit ($u_{n}$) la suite définie, pour tout nombre entier naturel $n$, 
par $u_{n} = 20\:000 - a_{n}$.\\
\textbf{a)} Montrer que la suite ($u_{n}$) est une suite géométrique, dont on 
précisera la raison et le premier terme.\\ 
\textbf{b)} Soit $n$ un nombre entier naturel ; exprimer $u_{n}$ en fonction 
de $n$. En déduire que, pour tout nombre entier naturel $n$, on a $a_{n} = 
20\:000 - 13\:000 \times 0,8^n$.\\
\textbf{c)} En utilisant le résultat précédent, déterminer la 
limite de la suite ($a_{n}$).\\
\textbf{d)} Après combien d'années le nombre d'abonnés dépassera-t-il 
16\:000 ?\\ 
 
\noindent \textbf{Problème}\hspace{2cm}11 points\\
 
Les objectifs de ce problème sont, en s'appuyant sur une fonction 
auxiliaire (partie A), l'étude d'une fonction $f$, le tracé de sa 
représentation graphique et le calcul d'une aire associé (partie B).\\ 
 
\noindent \textbf{Partie A}\\
$\bigstar~$\textbf{Étude d'une fonction auxiliaire}\\
 
Soit $g$ la fonction numérique définie pour tout $x$ appartenant à 
l'intervalle ]0 ; + ~$\infty$[ par $g(x) = x^2 - 2 + \ln x$.\\
 
\noindent \textbf{1)} Étudier le sens de variation de $g$ (on ne demande pas
d'étudier les limites de $g$ en 0 et en +~$\infty$).\\ 
 
\noindent \textbf{2)} Montrer que l'équation $g(x) = 0$ a une solution unique,
notée $\alpha$ appartenant à l'intervalle [1 ; 2].\\ 
Expliquer pourquoi $\alpha$ est la seule solution de l'équation $g(x) = 0$, 
pour $x$ appartenant à ]0 ; + ~$\infty$[.\\
Donner un encadrement de $\alpha$, d'amplitude $10^{-2}$.\\
 
\noindent $\triangleright~$\textbf{3)} Étudier le signe de $g(x)$ en 
fonction de $x$.\\
 
\noindent \textbf{Partie B}\\
$\bigstar~$\textbf{Étude d'une fonction \boldmath$f$\unboldmath et calcul d'une
aire}
 
On considère la fonction $f$ définie sur ]0 ; + ~$\infty$[ par $f(x) = 
x + \cfrac{ 1 - \ln x}{x}$ et on note $f'$ sa dérivée.\\
Soit $\mathcal{C}$ la courbe représentative de $f$ dans le plan 
rapporté à un repère orthonormal (O ; ~$\overrightarrow{\imath} , 
~\overrightarrow{\jmath}$ ) (unité graphique : 2 cm).\\ 
 
\noindent $\triangleright~$\textbf{1)} Étudier les limites de $f$ aux 
bornes de son ensemble de définition.\\
 
\noindent $\triangleright~$\textbf{2)} Calculer $f'(x)$ et vérifier que 
pour tout $x$ appartenant à ]0 ; + ~$\infty[,~ f'(x) = \cfrac{g(x)}{x^2}$.
En déduire le sens de variation de $f$.\\ 
 
\noindent $\triangleright~$\textbf{3) a)} Montrer que la droite 
($\Delta$) d'équation $y = x$ est asymptote à $\mathcal{C}$.\\
\textbf{b)} Déterminer le point d'intersection I de $\mathcal{C}$ et ($\Delta$)
et étudier la position de $\mathcal{C}$ par rapport à ($\Delta$).\\ 
 
\noindent $\triangleright~$\textbf{4)} Tracer la droite ($\Delta$) et la 
courbe $\mathcal{C}$.\\ 
 
\noindent $\triangleright~$\textbf{5)} Calculer la valeur exacte de 
l'aire, en cm$^2$, de la partie comprise sur le graphique entre $\mathcal{C}$,
($\Delta$) et les droites d'équations respectives $x = 1$ et $x $~= e.\\
(On pourra remarquer que $\cfrac{\ln x}{x} = \cfrac{1}{x} \times \ln 
x).$ \footnote{\scalebox{1 -1}{Nouvelle-Calédonie décembre 1998}}\\
 \newpage
%%%%  Sujet 11 Polynésie, session de remplacement %%%%
%%%%               septembre 1998                 %%%%
\noindent  \doublebox{TES   
\hspace{1.9cm}\Large{\textbf{Baccalauréat septembre 1998}} 
\hspace{1.9cm}\normalsize{}}\\ 
 
 \noindent \textbf{Exercice 1}\hspace{2cm}4 points\\
 
Le tableau ci-dessous représente la dette extérieure en pourcentage 
du PIB pour la Belgique (PIB : Produit Intérieur Brut). 
\begin{center} \begin{tabular}{| p{1,5cm} *{11}{| c} |}\hline
$x_{i}$~: années &1975& 1976& 1977 &1978& 1979& 1980& 1981& 1982& 
1983& 1984& 1985\\ \hline 
$y_{i}$~ dette en \% du PIB & 0,2& 0,1 &0,1 &   0,5 &  1,8 &  4,5 &   11,0& 16,6&
20,1& 23,2& 21,2\\ \hline
\end{tabular}\\
\end{center}
\begin{flushright} \small{Source : CEE Eurostat Monnaies et Finances 
1987.} \end{flushright}
 
\noindent $\triangleright~$\textbf{1)} Représenter le nuage de points 
associés à cette série statistique en choisissant des unités graphiques 
adaptées.\\
 
\noindent $\triangleright~$\textbf{2)} Donner le c¦fficient de 
corrélation linéaire de cette série statistique : le
résultat sera lu sur la calculatrice et arrondi à $10^{-2}$ près.\\
 
\noindent $\triangleright~$\textbf{3)} On veut déterminer la droite 
de régression de $y$ en $x$ obtenue par la méthode des moindre carrés. 
Caractériser cette droite par une équation de la forme $y = mx + p$$m$ est l'arrondi à $10^{-4}$ près et $p$ l'arrondi à $10^{- 1}$ près des
valeurs lues sur la calculatrice.\\
 
\noindent $\triangleright~$\textbf{4) a)} Utiliser la question 
précédente pour prévoir la dette extérieure de la Belgique, en pourcentage
du  PIB en 1986.\\
\textbf{b)} La valeur réelle atteinte en 1986 est égale à 20,6. 
\'A quelle erreur, en pourcentage de la valeur réelle, l'estimation 
conduit-elle  ?\\
 
\noindent \textbf{Exercice 2}\hspace{2cm}5 points\\
\textbf{(obligatoire)}\\
 
Un patineur participe à une compétition. Deux de ses sauts 
l'inquiètent. Il ne réussit le premier saut que dans 95 \% des cas. Comme il est
émotif, s'il ne réussit pas ce premier saut, il rate le deuxième 3 fois sur
10 ; sinon, si tout va bien lors du premier saut, il réussit le deuxième dans
90 \% des cas.\\
On notera $\overline{\textrm{A}}$ l'événement contraire d'un événement 
A.\\
Soit R$_{1}$ l'événement : « le patineur réussit le premier saut ».\\
Soit R$_{2}$ l'événement : « le patineur réussit le deuxième saut ».\\
 
\noindent $\triangleright~$\textbf{1) a)} Calculer la probabilité de 
l'événement R$_{1}$.\\
\textbf{b)} Calculer la probabilité de l'événement  sachant que 
R$_{1}$ est réalisé.\\
\textbf{c)} Calculer la probabilité de l'événement R$_{2}$ sachant que 
R$_{1}$ n'est pas réalisé.\\
 
\noindent $\triangleright~$\textbf{2)} Déterminer la probabilité de 
l'événement : « le patineur réussit les deux sauts ».\\ 
 
\noindent $\triangleright~$\textbf{3) a)} Calculer la probabilité de 
l'événement R$_{2}$.\\
\textbf{b)} Un spectateur, arrivé en retard, voit le patineur réussir le 
deuxième saut. Calculer la probabilité qu'il ait aussi réussi le premier 
saut.\\
 
\noindent $\triangleright~$\textbf{4)} Manquer le premier saut fait 
perdre 0,1 point, manquer le deuxième saut fait perdre 0,2 point ; le 
règlement prévoit que les pénalités s'ajoutent.\\ 
Soit $X$ la variable aléatoire donnant le total des pénalités obtenues 
par ce patineur lors de la compétition.\\ 
\textbf{a)} Déterminer la loi de probabilité de $X$.\\
\textbf{b)} Calculer l'espérance mathématique de $X$. Quelle interprétation 
peut-on en faire ?\\
 
\noindent \textbf{Problème}\hspace{2cm}11 points\\
 
Le plan est rapporté à un repère orthonormal (O ; 
$\overrightarrow{\imath},~\overrightarrow{\jmath})$. On prendra pour
unité graphique 1 cm sur chaque axe. Soit $f$ la fonction définie 
sur $\R$ par :
$$f(x) = (2,5 + x) \textrm{e}^{-0,5x + 1}.$$
\noindent \begin{center} \textbf{Partie A} \end{center}
\noindent $\triangleright~$\textbf{I)} Étude de la fonction $f$.\\ 
 
\noindent $\triangleright~$\textbf{1)} Étudier le sens de variation 
de $f$. 
 
\noindent $\triangleright~$\textbf{2)} Calculer $\lim \limits_{x \to 
-~\infty} f(x).$ 
 
\noindent $\triangleright~$\textbf{3)} Vérifier que pour tout réel 
$x,~ f(x) = \cfrac{2,5}{\textrm{e}^{-0,5x + 1}} + 2\textrm{e} \times 
\cfrac{ 0,5x}{\textrm{e}^{-0,5x}}.$\\
En déduire $\lim\limits_{x \to +~\infty}\: f(x).$\\ 
 
\noindent $\triangleright~$\textbf{4)} Dresser le tableau de variation de
$f$. Tracer sa représentation graphique dans le repère (O ; 
$\overrightarrow{\imath},~\overrightarrow{\jmath})$.\\
 
\noindent $\triangleright~$\textbf{II} Soit $g$ la fonction définie 
sur $\R$ par $g(x) =  0,3x + 1$.\\ 
 
\noindent $\triangleright~$\textbf{1)} Construire dans le même repère 
(O ; ~$\overrightarrow{\imath},~\overrightarrow{\jmath})$ la représentation 
graphique de $g$.\\ 
 
\noindent $\triangleright~$\textbf{2)} On veut résoudre dans l'intervalle [0; 10]
l'équation $f(x) = g(x)$, c'est-à-dire $f(x) - g(x) = 0$.\\ 
Pour cela on pose, pour tout $x$ de [0 ; 10], $h(x) = f(x) - g(x)$.\\
\textbf{a)} En utilisant les résultats obtenus à la question 
\textbf{1)}, montrer que, pour tout $x$ de [0 ; 10], ~$h'(x)$ est strictement
négatif.\\
\textbf{b)} En déduire que l'équation $h(x) = 0$ admet dans [0 ; 101 une
solution unique que l'on notera $\alpha$.\\
\textbf{c)} Par lecture graphique, encadrer $\alpha$ à l'aide de deux nombres 
entiers consécutifs.\\
\textbf{d)} Donner un encadrement de $\alpha$ d'amplitude $10^{-2}$. 
 
\noindent \begin{center} \textbf{Partie B} \end{center}
 
\noindent $\triangleright~$\textbf{I)} On considère un produit pour 
lequel, en fonction du prix unitaire $p$ (en francs), la demande est donnée par
$f(p)$ et l'offre par $g(p)$ ($p$ appartient à [0 ; 10]).\\
 
\noindent $\triangleright~$\textbf{1)} Donner le prix d'équilibre 
c'est-à-dire celui pour lequel l'offre est égale à la demande.\\
 
\noindent $\triangleright~$\textbf{2)} Vérifier que, pour un prix de 
3,10 F, si le prix augmente de 1 \%, la demande diminue de 1 \% 
environ.\\
 
 
\noindent $\triangleright~$\textbf{II)} La fonction E est définie sur 
l'intervalle [0 ; 101 par E($x) = x \cfrac{f'(x)}{f(x)}$.\\
En économie, E désigne l'élasticité de $f$.\\ 
 
\noindent $\triangleright~$\textbf{1)} Vérifier que, pour tout $x$ de 
[0 ; 10], E($x) = \cfrac{ -~ 0,5x^2 - 0,25x}{x + 2,5}$.\\ 
 
\noindent $\triangleright~$\textbf{2) a)} Résoudre dans [0 ; 101 
l'équation : E$(x)$ = - 1.\\
\textbf{b)} Donner une valeur décimale approchée à $10^{-2}$ par défaut de
la solution.\\
 \footnote{\scalebox{1 -1}{Polynésie session de remplacement,  septembre 1998}}\\
 \newpage
 %%%%%%%%  Sujet 12 Sportifs de haut-niveau  %%%%%%%
 %%%%%%%%  octobre 1998   %%%%%%%%%%%%
 \noindent  \doublebox{TES   \hspace{1.9cm}\Large{\textbf{Baccalauréat octobre 1998}} 
\hspace{1.9cm}\normalsize{}}\\
 
 \noindent \textbf{Exercice 1}\hspace{2cm}4 points\\
 
Tous les résultats seront donnés sous forme de fractions 
irréductibles.\\
Neuf amis, cinq garçons et quatre filles, décident de tirer au sort 
deux conducteurs, qui devront rester sobres durant une soirée.\\
Chacun écrit son nom sur un carton glissé ensuite dans une boîte.
L'un d'entre eux extrait au hasard, successivement et sans remise, 
deux cartons de la boîte.\\
On définit les événements G$_{1}$,~ G$_{2}$,~ F$_{1}$, et F$_{2}$ par
G$_{1}$ « Un garçon est désigné au premier tirage » ;\\
G$_{2}$ « Un garçon est désigné au deuxième tirage » ;\\
F$_{1}$ « Une fille est désignée au premier tirage » ;\\
F$_{2}$ « Une fille est dvsignée au deuxième tirage ».\\
 
\noindent $\triangleright~$\textbf{1) a)} Calculer la probabilité que 
le nom d'une fille apparaisse au deuxième tirage sachant que le nom 
d'un garçon a été lu sur le premier carton.\\
\textbf{b)} Calculer la  probabilité de l'événement G$_{1}~ \cap~$  
F$_{2}$.\\ La comparer à celle de l'événement G$_{2}~ \cap~ $ F$_{1}$.\\ 
 
\noindent $\triangleright~$\textbf{2)} Calculer la probabilité qu'il 
y ait deux conductrices en fin de soirée.\\ 
 
\noindent $\triangleright~$\textbf{3)} Calculer la probabilité que le 
sort désigne une fille au deuxième tirage.\\ 
 
\noindent $\triangleright~$\textbf{4)} Soit $X$ la variable aléatoire 
égale au nombre de filles désignées.\\
\textbf{a)} Déterminer la loi de probabilité de $X$.\\
\textbf{b)} Calculer son espérance mathématique E($X$).\\ 
 
 
\noindent \textbf{Exercice 2} \hspace{3cm}5 points\\
\textbf{(obligatoire)}\\
 
Dans le plan rapporté à un repère orthonormal (unité graphique : 4 
cm), la courbe $\mathcal{C}$, représentée ci-dessous représente une fonction
$f$ définie et dérivable sur l'intervalle I = ]0 ; ~e$^{1,5}$].\\
La fonction dérivée de $f$ est notée $f'$.\\
Les variations de $f$ sont données par le tableau suivant :
\begin{center} \begin{pspicture}(5,3)
\rput(0.5,2.5){$x$} \rput(1.2,2.5){$0$} \rput(3,2.5){$a$}  \rput(4.6,2.5){e$^{1,5}$} 
\rput(0.6,1){$f(x)$} \psline(1.1,0)(1.1,2)  \psline(1.2,0)(1.2,2) 
\rput(3,1.7){$1/4$} \psline{->}(1.5,0.1)(2.7,1.8) 
\psline{->}(3.3,1.8)(4.8,0.2)
 \psline(0,3)(5,3)  \psline(0,2)(5,2)  \psline(0,0)(5,0)
  \psline(0,0)(0,3)  \psline(1,0)(1,3)  \psline(0,0)(0,3)  \psline(5,0)(5,3)  
\end{pspicture}
\end{center}
On précise que :\\ 
$\bullet~$ Les droites ($\Delta$) et (D) sont tangentes à la courbe $\mathcal{C}$
respectivement aux points A d'abscisse $a$ et B d'abscisse 1.\\
$\bullet~$  La droite ($\Delta$) est parallèle à l'axe des abscisses.\\
$\bullet~$ L'axe des ordonnées est asymptote à  $\mathcal{C}$.\\ 
 
\noindent \textbf{I)} Par lecture graphique, sans justification des résultats,
donner : 
 
\noindent $\triangleright~$\textbf{1)} Les valeurs suivantes : 
$f($e),~ $f(a),~ f'(1),~ f'(a)$.\\
 
\noindent $\triangleright~$\textbf{2)} La limite de $f$ en 0.\\
 
\noindent $\triangleright~$\textbf{3)} Le signe de $f(x)$ selon les 
valeurs de $x,~ x$ étant dans l'intervalle I.\\
 
\noindent $\triangleright~$\textbf{4)} L'ensemble des solutions, sur 
l'intervalle I, de l'inéquation : $f'(x) ¾ 0$.\\
 
\noindent $\triangleright~$\textbf{5)} Une interprétation du nombre 
$\int\limits_{1}^{\textrm{e}} f(x)\: \textrm{d}x$ et trouver parmi les
intervalles suivants celui auquel appartient ce nombre :
$$[0~ ;~ 0,2[,~ [0,2~  ;~ 0,4[,~ [0,4~ ;~ 0,6[,~ [0,6~ ;~ 1[,~ [1~ ;~ 2[.$$
 
\noindent \textbf{II)} La fonction $f$ est définie sur 
]0 ; e$^{1,5}$] par : $f(x) = \ln x - (\ln x)^2$.\\
 
\noindent $\triangleright~$\textbf{1)} Retrouver par le calcul le résultat
trouvé en \textbf{I) 3)}.\\ 
 
\noindent $\triangleright~$\textbf{2)} Déterminer le nombre $a$, 
abscisse du point A de la courbe $\mathcal{C}$.\\
\psset{xunit=2cm,yunit=2cm}
\begin{center} \begin{pspicture}(-0.25,-1.25)(4.75,1.25)
    \psgrid[subgriddiv=4](0,0)(-0.25,-1.25)(5,1.25)
    \psline{->}(-0.25,0)(4.25,0) \psline{->}(0,-1.25)(0,1.25)
    \psline{<->}(0,-1)(2.25,1.25) \psline{<->}(0.5,0.25)(2.5,0.25) 
    \psline[linestyle=dashed](1.61,0)(1.61,0.25)
    \psline[linestyle=dashed](4.5,0)(4.5,-0.75)
    \rput(0.8,0.1){B}  \rput(1.61,0.4){A}  \rput(2.25,0.3){$\Delta$}
    \rput(4.5,0.1){e$^{1,5}$} 
     \rput(1.61,-0.2){$a$}  \rput(2.71,-0.2){e}  \rput(2.25,1.1){(D)}
     \rput(4,-0.72){$\mathcal{C}$}
     \psplot[plotpoints=1000]{0.5}{4.5}{ x ln x ln dup mul  sub}
\end{pspicture}
\end{center}
 
 
\noindent \textbf{Exercice 2} \hspace{3cm}5 points\\
\textbf{(spécialité)}\\
 
Un salarié remarque qu'il lui reste, chaque mois, 2000 F (francs 
français) de son salaire mensuel.\\
Il décide donc, en 1998, de réaliser une épargne « prudente » de la 
façcon suivante :\\
Le 28 de chaque mois, il verse 50 \% du solde de son compte courant 
sur un plan d'épargne.\\
Le solde est nul le 28 décembre 1997.\\
Le 28 janvier 1998, le solde de son compte courant est : S$_{1}$ = 2000 
F ; il verse donc la somme e$_{1}$ = 1000 F sur son plan d'épargne et laisse
1000 F sur son compte courant.\\
Le 28 février 1998, le solde S$_{2}$ est égal à 3000 F : c'est-à-dire 
1000 F restant, plus 2 000 F d'économies mensuelles. Il verse donc 
e$_{2}$ = 1500 F sur son plan d'épargne.\\
 
\noindent $\triangleright~$\textbf{1)} Calculer e$_{3}$ et e$_{4}$, versements 
respectifs de son compte courant à son plan d'épargne le 28 mars et le 
28 avril.\\
 
\noindent $\triangleright~$\textbf{2)} On désigne par e$_{n}$ le montant 
théorique du versement du compte courant au plan d'épargne le 28 du 
$n^{\textrm{e}}$  mois qui suit le mois de décembre 1997.\\
On a donc e$_{n+1} = \cfrac{1}{2}$(e$_{n} + 2000)$.\\
Pour tout nombre entier naturel $n$ non nul, on définit la suite 
($v_{n})$ par $v_{n} = 2000$ - e$_{n}$.\\
\textbf{a)} Démontrer que la suite ($v_{n}$) est géométrique de raison 0,5
et de premier terme $v_{1}$ = 1000.\\
\textbf{b)} En déduire l'expression de $v_{n}$ en fonction de $n$.\\
\textbf{c)} Calculer $v_{1} + v_{2} + \ldots  + v_{12}$.\\
 
\noindent $\triangleright~$\textbf{3) a)} Exprimer e$_{n}$ en fonction de 
$n$.\\
\textbf{b)} Trouver le montant de la somme capitalisée sur le plan d'épargne au
29 décembre 1998.\\
 
\noindent \textbf{Problème} \hspace{3cm}11 points\\
 
On considère un produit dont le prix unitaire est $x$ (en milliers de 
francs français).\\
D'après une étude de marché, l'offre $f(x)$ et la demande $g(x)$ (en 
milliers d'objets) de ce produit sont définies, pour tout $x$ positif ou nul, par 
les formules :
$$f(x) = \textrm{e}^{0,5x} - 1~ \textrm{et}~ g(x) = \cfrac{ 8}{\textrm{e}^{0,5x}
+ 1}$$
 
\noindent \textbf{Partie A}\\
 
\noindent $\triangleright~$\textbf{1) a)} Déterminer $f(0)$ et la 
limite de $f$ en + ~$\infty$.\\
\textbf{b)} Étudier les variations de $f$ sur [0 ; +~$\infty$[. 
 
\noindent $\triangleright~$\textbf{2) a)} Déterminer $g(0)$ et  la 
limite de $g$ en + ~$\infty$.\\
\textbf{b)} Étudier les variations de $g$ sur [0 ; +~ $\infty$[.\\
 
\noindent $\triangleright~$\textbf{3)} Le plan est rapporté à un 
repère orthonormal (on prendra pour unité graphique 4 cm).\\
Tracer les courbes représentatives des fonctions $f$ et $g$ après avoir 
déterminé et tracé les tangentes respectives à ces deux courbes aux points 
d'abscisse 0.\\
 
\noindent \textbf{Partie B}\\
 
L'équation $f(x) = g(x)$ admet une solution unique $p$ dans l'intervalle 
[0 ~;~+~$\infty$[.\\
\noindent $\triangleright~$\textbf{1)} Par lecture graphique, donner une
approximation à 0, 1 près de $p$ et du nombre $n = f(p)$ (on fera 
apparaître les tracés permettant cette lecture).\\ 
 
\noindent $\triangleright~$\textbf{2) a)} Calculer $p$ et $n$.\\
\textbf{b)} Le nombre $p$ est appelé « prix d'équilibre » du produit.\\
Donner le prix d'équilibre, exprimé en francs, arrondi au franc près, 
ainsi que le nombre correspondant d'objets proposés sur le marché.\\
 
\noindent \textbf{Partie C}\\
 
On considère les nombres I = $\int\limits_{0}^{\ln 9}\: 
g(x)\:\textrm{d}x$ et J	 = I - $np$.\\
 
\noindent $\triangleright~$\textbf{1)} Donner une interprétation 
géométrique de I.\\ 
En déduire une interprétation géométrique de J (on pourra utiliser 
des hachures de couleurs différentes).\\ 
 
\noindent $\triangleright~$\textbf{2)} Soit $h$ la fonction définie sur 
l'intervalle [0 ; + ~$\infty$[ par :
$$h(x) = x - 2 \ln \left(\textrm{e}^{0,5x} + 1\right).$$
Déterminer $h'(x)$ où $h'$ désigne la fonction dérivée de $h$.\\
\textbf{b)} En déduire que I égale $8 \ln 9 - 8 \ln 4$.\\
\textbf{c)} En économie, on considère que J exprime, en millions de francs, la 
« rente » des consommateurs.\\
Déterminer, au millier de francs près, une estimation de la « rente » des
consommateurs.\\
\footnote{\scalebox{1 -1}{Sportifs de haut-niveau octobre 1998}}
\end{document}